You are on page 1of 602
Mark this question => Question Td : 48883 Question 1 of 30 4.38 year old woman present to you convinced that she has chronic Fatigue syndrome. No history of fever or swollen glands is present, and no other symptoms other than fatigue and difficulty sleeping is present, Her physical exemination is completely normal She has a high TSH level, You would then suspect and subsequently confirm the diagnosis of which of the following? 2) Hyperthyroidism secondary to pituitary adenoma 6) Grave disease ©) Hypothyroidism 4) Thyrcicltis ©) Chronic fatigue syndrome Anower [UEIRIRISHONY) Other Users Explanation Repost An Exot Question Explanation: Hypothyroidism is underactvity of the thyroid gland that leads to inadequate production of thyroid hormone and a slowing of vital tbody functions. Insufficient thyroid hormone causes body functions to slow. Symptoms are subile and develop gradually. They may bbe mistaken for depression, especially among older people. Facial expressions become dull, the voice is hoarse and speech is slow, eyelids droop, and the eyes and face become puffy. Many people with hypothyroidism gain weight, become constipated, and are unable to tolerate cold. Hypothyroidism is common, especially among older people, particularly women: it affects about 10% of older women. It can, however, occur at any age. Very severe hypothyroidism is called myxedema Diagnosis is made by obiaining TSH and Free T4 levels. Treatment is with thyrcid replacement with synthroid. Mark this question => Question Td : 48883 Question 1 of 30 A 38 year old woman present to you convinced that she has chronic fatigue syndrome. No history of fever or swollen glands is presert, and no other symptoms other than fatigue and difficulty sleeping is present, Her physical exemination is completely nonmal She has a high TSH level, You would then suspect and subsequently confimn the diagnosis of which of the following? 2) Hyperthyroidism secondary to pituitary adenoma ) Grave disease V © 0) Hypothyroidism 4) Thyreicltis «) Chronic fatigue ayndrome Anower [UEIRIRISHONY) Other Users Explanation Repost An Exot Question Explanation: Hypothyroidism is underactvity of the thyroid gland that leads to inadequate production of thyroid hormone and a slowing of vital tbody functions. Insufficient thyroid hormone causes body functions to slow. Symptoms are subile and develop gradually. They may bbe mistaken for depression, especially among older people. Facial expressions become dull, the voice is hoarse and speech is slow, eyelids droop, and the eyes and face become puffy. Many people with hypothyroidism gain weight, become constipated, and are unable to tolerate cold. Hypothyroidism is common, especially among older people, particularly women: it affects about 10% of older women. It can, however, occur at any age. Very severe hypothyroidism is called myxedema Diagnosis is made by obiaining TSH and Free T4 levels. Treatment is with thyrcid replacement with synthroid. 2242014 2:46:27 PM Mark this question & => Question Td : 50519 Question 2 of 30 A.50 year cld male who smokes two packs of cigarettes a day presents with a hing mass on K-ray and recent weight gain. Laboratory investigation shows hypanctremia with hyperosmoler urine. The mast liceiy diagnosis in this patient is 2) Renal failure +) Pituitary Eaibure ©) Conn’s syndrome 4) Inappropriate ADH (cecretion) 6) Cardiac failure Answer | Explanation Other User's Explanation Report An Error Question Explanation: SLADH (syndrome of inappropriate secretion of anti-dinretic hormone) is a common paraneoplastic syndrome that affects the endocrine system. This syncrome is most offen associated with strall cell ling cancer; however, other cancers such as brain tumors, eukemia, lymphoma, colon, prosiate, and head and neck cancers can lead to SIADH. SIADHT is caused by the ineppropriate production and secretion of arginine vasopressin ot antidiuresic hoomone (ADH) by tumor cells, Patients with SIADH may not have syinptoms, especially i the early stages. When symptoms do occur they are usually related to hyponatremia, which leads to central nervous system toxicity ifleft untreated Lab values will show concentrated urine in the presence of plasma hypo-osmolaity (hyponatremia), Signs and syinptomns associated with hyponetremia include fatigue, anorexia, headache and mild alteration in mental status in early stages IESIADH remains untreated, symptoms can progress to confision, delirium, seimres, coma, and death. Treatment approaches for SIADH ore to treat the underlying tumor and restriction of fluids. More severe cases may require the administration of medications 2242014 2:46:27 PM Mark this question & => Question Td : 50519 Question 2 of 30 A.50 year cld male who smokes two packs of cigarettes a day presents with a hing mass on K-ray and recent weight gain. Laboratory investigation shows hypanctremia with hyperosmoler urine. The mast liceiy diagnosis in this patient is 2) Renal failure +) Pituitary Eaibure ©) Conn’s syndrome Y © 4) lnappropriste ADH (secretion) 6) Cardiac failure Question Explanation: SLADH (syndrome of inappropriate secretion of anti-dinretic hormone) is a common paraneoplastic syndrome that affects the endocrine system. This syncrome is most offen associated with strall cell ling cancer; however, other cancers such as brain tumors, eukemia, lymphoma, colon, prosiate, and head and neck cancers can lead to SIADH. SIADHT is caused by the ineppropriate production and secretion of arginine vasopressin ot antidiuresic hoomone (ADH) by tumor cells, Patients with SIADH may not have syinptoms, especially i the early stages. When symptoms do occur they are usually related to hyponatremia, which leads to central nervous system toxicity ifleft untreated Lab values will show concentrated urine in the presence of plasma hypo-osmolaity (hyponatremia), Signs and syinptomns associated with hyponetremia include fatigue, anorexia, headache and mild alteration in mental status in early stages IESIADH remains untreated, symptoms can progress to confision, delirium, seimres, coma, and death. Treatment approaches for SIADH ore to treat the underlying tumor and restriction of fluids. More severe cases may require the administration of medications Report An Error 2124/2014 2:46:41 PM “Mark this question €&-_> Question 3 of 30 A.72 year old woman comes t» you for the first te, She has taken levothyroxine (Synthroid), 0.3 mg/day, for the last 20 years Although a recent screening TSH was filly suppressed at <0.1 micro UimL, she claims that she has fe ‘awful’ when previous physician have attempted to lower hor dosage. You explain that a serious potential complication of her current thyroid medication is ‘which of the following? a) Adrenal insufficiency b) Carcinoma ofthe ovary ©) Carcinoma of the thyroid 4) Renal faiure ©) Hip fracture newer [FESINRHAN) othe: Users Explanation Repost An Esor Question Explanation: ‘Women older than 65 years of age who have low serum TSH levels, indicating physiologic hyperthyroidism, are at increased risk for new hip and vertebral actures, Use of thyroid hormone itself does not increase the risk of fracture if TSH levels are normal 2124/2014 2:46:41 PM “Mark this question €&-_> Question 3 of 30 A.72 year old woman comes t» you for the first te, She has taken levothyroxine (Synthroid), 0.3 mg/day, for the last 20 years Although a recent screening TSH was filly suppressed at <0.1 micro UimL, she claims that she has fe ‘awful’ when previous physician have attempted to lower hor dosage. You explain that a serious potential complication of her current thyroid medication is ‘which of the following? 8) Adrenal insufficiency +b) Carcinoma of the ovary ©) Carcinoma of the thyroid 4) Renal faiure Y © ©) Hip fracture newer [FESINRHAN) othe: Users Explanation Repost An Esor Question Explanation: ‘Women older than 65 years of age who have low serum TSH levels, indicating physiologic hyperthyroidism, are at increased risk for new hip and vertebral actures, Use of thyroid hormone itself does not increase the risk of fracture if TSH levels are normal 2/24/2014 2:46:59 PM “Mask this question => Question Td : 51659 Question 4 of 30 (Out of the following, which is associeted with thyroid disease? a) Newrofibromatosis b) Vitiligo ) Erythema nodosum 4) Pemphigus wilgans 6) Iethyosis mulgaris Answer | Bevlanation Other User's Explanation Report An Error Question Explanation: ‘Vitiligo is a loss of skin melanocytes that causes areas of skin depigmentation of varying sizes. Cause is known, but the condition may be autoimmune, ac up to 1/3 of patients have evidence of other eutoimmane disease. Vitlige affects 0.5 t> 2% of the population Biiology is unknown, but melanocytes are lacking in affected areas, some patients have antibodies to melanin. Up to 30% have other autoimmune anibodies (te thyroglobulin, adrenal cells, and parietal cells) or clinical autoimmune endocrinopathies (Addison's disease, diabetes mellitus, petnicious anemia, and thyroid dysfunction), leading to speculation that vitiligo is an autoimmune disease However, the relationship is unclear and may be coincidental. The strongest association is with hyperthyroidisin (Gxaves' disease) and hypothyroidism (Hashimoto s thyrcidtis). 2/24/2014 2:46:59 PM “Mask this question => Question Td : 51659 Question 4 of 30 (Out of the following, which is associeted with thyroid disease? a) Newrofibromatosis Y © b) Vitiligo: ) Erythema nodosum 4) Pemphigus vulgaris ¢) Icthyosis vulgaris Answer | Bevlanation Other User's Explanation Report An Error Question Explanation: ‘Vitiligo is a loss of skin melanocytes that causes areas of skin depigmentation of varying sizes. Cause is known, but the condition may be autoimmune, as up to 1/3 of patients have evidence of other autoimmune disease. Vitligo affects 0.5 to 2% of the population. Biiology is unknown, but melanocytes are lacking in affected areas, some patients have antibodies to melanin. Up to 30% have other autoimmune antibodies (te thyroglobulin, adrenal cells, and parietal cells) or clinical autoimmune endocrinopathies (Addisons disease, diabetes mellitus, petnicious anemia, and thyroid dysfunction), leading to speculation that vitiligo is an autoimmune disease However, the relationship is unclear and may be coincidental. The strongest association is with hyperthyroidism (Graves’ disease) and hypothyroidism (Hashimoto ‘s thyrcidtis). 2i24/2014 2:47:26 PM Mark this question & => Question Id : 55763 Question 5 of 30 A 22 year old woman presents with complaints of heat intolerance, sleeplessness, nervousness and weight loss despite an excellent appetite. What changes in TSH and free Td would you expect? a) Decreased TSH, increased free T4 ‘b) Increased TSH, increased free T4 c) Decreased TSH, normal free T4 d) Increased TSH, decreased free T4 ) Normal TSH, increased free T4 Question Explanation: ‘Hyperthyroidism is characterized by hypermetabolism and elevated serum levels of free thyroid hormones. Symptoms are many but fnchde tachycardia, fetigne, weight loss, and tremor. Diagnosis is based on history. physical examination, and thyroid fanction tests, Serum TSH is the best test, because TSH is suppressed in hyperthyroid patients except when the etiology ic a TSH-secreting pituitary adenoma or pituitary resistance to thyroid hormone, Free T4 is increased. Report An Error 2i24/2014 2:47:26 PM Mark this question & => Question Id : 55763 Question 5 of 30 A 22 year old woman presents with complaints of heat intolerance, sleeplessness, nervousness and weight loss despite an excellent appetite. What changes in TSH and free Td would you expect? v a) Decreased TSH, increased free T4 ‘b) Increased TSH, increased free T4 c) Decreased TSH, normal free T4 d) Increased TSH, decreased free T4 ) Normal TSH, increased free T4 Question Explanation: ‘Hyperthyroidism is characterized by hypermetabolism and elevated serum levels of free thyroid hormones. Symptoms are many but fnchde tachycardia, fetigne, weight loss, and tremor. Diagnosis is based on history. physical examination, and thyroid fanction tests, Serum TSH is the best test, because TSH is suppressed in hyperthyroid patients except when the etiology ic a TSH-secreting pituitary adenoma or pituitary resistance to thyroid hormone, Free T4 is increased. Report An Error 2i24/2014 2:47:42 PM Mark this question & => Question Ie : 55813 Question 6 of 30 A 22 year old gil reports that she has been troubled by episodes during which she feels apprehensive and which usvally occur in the ‘moming. Her heart rate increases. She sweats excessively. Agitation and restlessness are prominent. The appropriate laboratory tests to be ordered are a) Thyroid fimncton tests b) Toxicolegic screen c) Measurement of serum sodium level 4) Measurement of serum glucose level ¢) Measurement of serum ammonia level Question Explanation: ‘Hyperthyroidism is characterized by hypermetabolsm and elevated serum levels of ffee thyroid hormones. Many common symptoms and signs of hyperthyroidism are similar to those of adrenergic excess, such es nervousness, palpitations, hyperactivity, increased sweating, heat hypersensitivity, fatigue, increased appetite, weight loss, insomnia, weakness, and frequent bowel movements (eccasionalyy diathea). Hypomenorhea may be present. Signs may include warm, moist skin, iremor, tachycardia, widened pulse pressure, atrial firilation and palptators, Diagnosis is based on history, physical examination, and thyroid function tests. Serum TSH is the best test, because TSHis suppressed in hyperthyroid patients except when the etiology is a TSH secreting pituitary adenoma or pititary resistance to thyrcid honnone. Frer T4 is increased, Report An Error 2i24/2014 2:47:42 PM Mark this question & => Question Ie : 55813 Question 6 of 30 A 22 year old gil reports that she has been troubled by episodes during which she feels apprehensive and which usvally occur in the ‘moming. Her heart rate increases. She sweats excessively. Agitation and restlessness are prominent. The appropriate laboratory tests to be ordered are Y © a) Thyroid fimction tests b) Toxicolegic screen c) Measurement of serum sodium level 4) Measurement of serum glucose level ¢) Measurement of serum ammonia level Question Explanation: ‘Hyperthyroidism is characterized by hypermetabolsm and elevated serum levels of ffee thyroid hormones. Many common symptoms and signs of hyperthyroidism are similar to those of adrenergic excess, such es nervousness, palpitations, hyperactivity, increased sweating, heat hypersensitivity, fatigue, increased appetite, weight loss, insomnia, weakness, and frequent bowel movements (eccasionalyy diathea). Hypomenorhea may be present. Signs may include warm, moist skin, iremor, tachycardia, widened pulse pressure, atrial firilation and palptators, Diagnosis is based on history, physical examination, and thyroid function tests. Serum TSH is the best test, because TSHis suppressed in hyperthyroid patients except when the etiology is a TSH secreting pituitary adenoma or pititary resistance to thyrcid honnone. Frer T4 is increased, Report An Error 2/24/2014 2:47:58 PM Mabie Gein e-= Question 7 of 30 A 32 year old woman complains of episodic faintness, tingling sensation in her hands, shortness of breath, and severe anxiety. ‘Thorough medical workup reveals no pathologic condition, Durng an episode of these symptoms chemical analysis of the serum will probably reveal a) Decreased chloride ) Increased wea (BUN) ©) Decreased protein 6) Increased pH @) Increased serum amylase Question Explanation: ‘A pheochromocytomais a catechokanine secreting tumor of chromaffin cells typically located in the adrenals. It causes persistent or paroxysmal hypertension. Common symptoms and signs are paroxysms of tachycardia, diaphoresis, postural hypotersion, tachypnea, cold and clammy skin, cevere headache, angina, palpitations, naucea, vomiting, epigastric pain, visual disturbances, dyspnea, paresthesias, conctipatica, and asense ofimpending doom. Diagnosis is by measuring catecholatrine psoducts in blood or urine, Tinaging tests, especially CT or MEL, help localize tumors. Treatment involves removal of the, tumor when possible. Drug therapy for control of BP includes alpha- blockade. possibly combined with B-blockade. Ectopic amylase production by lung, ovary. pancreas, and colon malignancies: pheochromocytoma, thymoma; rmiltiple reyeloma (increased calvary amylase); and breast cancer (ncreased pancreatic amylase) are miscellaneous causes of hyperanytasemia 2/24/2014 2:47:58 PM Mabie Gein e-= Question 7 of 30 A 32 year old woman complains of episodic faintness, tingling sensation in her hands, shortness of breath, and severe anxiety. ‘Thorough medical workup reveals no pathologic condition, Durng an episode of these symptoms chemical analysis of the serum will probably reveal a) Decreased chloride +) Increased wea (BUN) ©) Decreased protein 6) Increased pH v © 2) Increased serum amylase Question Explanation: ‘A pheochromocytomais a catechokanine secreting tumor of chromaffin cells typically located in the adrenals. It causes persistent or paroxysmal hypertension. Common symptoms and signs are paroxysms of tachycardia, diaphoresis, postural hypotersion, tachypnea, cold and clammy skin, cevere headache, angina, palpitations, naucea, vomiting, epigastric pain, visual disturbances, dyspnea, paresthesias, conctipatica, and asense ofimpending doom. Diagnosis is by measuring catecholatrine psoducts in blood or urine, Tinaging tests, especially CT or MEL, help localize tumors. Treatment involves removal of the, tumor when possible. Drug therapy for control of BP includes alpha- blockade. possibly combined with B-blockade. Ectopic amylase production by lung, ovary. pancreas, and colon malignancies: pheochromocytoma, thymoma; rmiltiple reyeloma (increased calvary amylase); and breast cancer (ncreased pancreatic amylase) are miscellaneous causes of hyperanytasemia 2124/2014 2:48:09 PM ‘Mark this question & => Question Id : 62690 Question 8 of 30 ‘True statement with respect to diabetes and kidney disease is 2) Primary affective the tubules ) Earliest sign is decreased GER. ©) Microalbuminuria is alate sign of DIM nephropathy 4) Thteshold for dialysic ie same as other CBF patierts ©) BP control slows progression of DM nephropathy Answer | Explanation Other User's Explanation Report An Error Question Explanation: Jn dabetics its important t» test the urine for protein, Blood tests can tell how much damage has been done to the kidneys, The most important things a patient can do to control the progression of their diabetes is t> keep blood pressure lower than 130 over 80 Keep HbAIC level below 7 percent and leading a balance lifestyle with dict and exercise. 2124/2014 2:48:09 PM ‘Mark this question & => Question Id : 62690 Question 8 of 30 ‘True statement with respect to dabetes and kidney disease is 2) Primary affective the tubules 't) Earliest sign is decreased GER ¢) Microalbuminuria is alate sign of DM nephropathy 4) Threshold for dialysis is same as other CRF patients Y © ©) BP control slows progression of DM nephropathy Answer | Explanation Other User's Explanation Report An Error Question Explanation: Jn dabetics its important t» test the urine for protein, Blood tests can tell how much damage has been done to the kidneys, The most important things a patient can do to control the progression of their diabetes is t> keep blood pressure lower than 130 over 80 Keep HbAIC level below 7 percent and leading a balance lifestyle with dict and exercise. 2242014 2:48:23 PM ‘Mark this question & => Question Id : 64396 Question 9 of 30 Low to xonnal follicle stimulating hormone level (FSH) is found in patients with which of the following? a) Resistant ovary syndrome ») Previous pelvic radiotherapy for cervical cancer ©) Anorestia nervosa. 4) Pure gonadal dysgenesis ) Premature ovarian failure Question Explanation: ‘A low calosic intake not only interferes with the nutritional needs ofa young and growing organism, but also interferes with the homeostatic mechanisms necessary to maintain functioning. Undernutrition and weight loss associated with anorexia nervosa either reverse pubertal changes and prevent menarche from occurring (orimery amerorshea) or regress the hyperthalamo-pituitary-gonadal avis to prepubertal regulation, resulting in low LE, low FSH and minimal estrogen and progesterone plasma levels (secondary amenorhhiea), 2242014 2:48:23 PM ‘Mark this question & => Question Id : 64396 Question 9 of 30 Low to normal folicle stimulating hormone level (FSH) is found in patients with which of the following? 4) Resistant ovary syndrome +) Previous pelvic radiotherapy for cervical cancer Y © ©) Anoresia nervosa © Pure gonadal dysgenesis ) Premature ovarian failure Question Explanation: ‘A low calosic intake not only interferes with the nutritional needs ofa young and growing organism, but also interferes with the homeostatic mechanisms necessary to maintain functioning. Undernutrition and weight loss associated with anorexia nervosa either reverse pubertal changes and prevent menarche from occurring (orimery amerorshea) or regress the hyperthalamo-pituitary-gonadal avis to prepubertal regulation, resulting in low LE, low FSH and minimal estrogen and progesterone plasma levels (secondary amenorhhiea), 2124/2014 2:48:37 PM Mark this question €=> Question 10 of 30 A 66 year old man presents to the emergency room with a four day history of worsening headaches and confusion, He is not taking medications. Physical examination confirms the confusion, but is otherwise normal. His plastna sodium is found to be 117 mmol/L. (aonmal 135 -145 mmol/L) and plasma osmolality is 248 mmollcg H20 (normal 285 -29Smmolfkeg H20). The most likely diagnosis is a) Hypoadtenalisen ) Congestive heart failure ¢) Pseudohyponatremia 4) Syndrome ofinappropriate antidiuretic hormone secretion €) Hyperthyroidsm Question Explanation: ‘A. cause of hyponatremia is the syndrome of inappropriate secretion of antidiuretic hormone (SLADH). People who have SIADH have a low sodium level because the pimitary gland secretes too much antidiuretic hormone. Hyponetremia also oceurs in people who kkave underactive adrenal glands and wha thu: excrete too much sadines in the urine. This can result in 2 low plasma osmelalty due ta the excess water being brought back into the bloodstream by the excess ADH. When the sodium leveliin the blood falls quickly, symptoms tend to develop rapicly and be more severe. The brains particularly sensitive to chenges in the sodium level in the blood. ‘Therefore, symptoms such as lethargy and confusion occur frst. As hyponatremia becomes more severe, muscle twitching and seinures may occur, stupor, comma, and death may follovr. The diagnosis of hyponatremia is made hy measuring the sodium level in the blood 2124/2014 2:48:37 PM Mark this question €=> Question 10 of 30 A 66 year old man presents to the emergency room with a four day history of worsening headaches and confusion, He is not taking medications. Physical examination confirms the confusion, but is otherwise normal. His plastna sodium is found to be 117 mmol/L. (aonmal 135 -145 mmol/L) and plasma osmolality is 248 mmollcg H20 (normal 285 -29Smmolfkeg H20). The most likely diagnosis is a) Hypoadtenalisen ) Congestive heart failure ¢) Pseudohyponatremia V © 4) Syndrome ofinappropsiate antidiuretic hormone secretion €) Hyperthyroidsm Question Explanation: ‘A. cause of hyponatremia is the syndrome of inappropriate secretion of antidiuretic hormone (SLADH). People who have SIADH have a low sodium level because the pimitary gland secretes too much antidiuretic hormone. Hyponetremia also oceurs in people who kkave underactive adrenal glands and wha thu: excrete too much sadines in the urine. This can result in 2 low plasma osmelalty due ta the excess water being brought back into the bloodstream by the excess ADH. When the sodium leveliin the blood falls quickly, symptoms tend to develop rapicly and be more severe. The brains particularly sensitive to chenges in the sodium level in the blood. ‘Therefore, symptoms such as lethargy and confusion occur frst. As hyponatremia becomes more severe, muscle twitching and seinures may occur, stupor, comma, and death may follovr. The diagnosis of hyponatremia is made hy measuring the sodium level in the blood 2124/2014 2:18:51 PM “Mark this question = Question Id : 64448 Question 11 of 30 A 35 year old laborer comes to the office because of 2 kg (5-Ib) weight loss and an increased appetite. He has diabetes melitus and has been taking insulin in divided doses. He says thathome monitoring of his serum glucose concentration has shows values from 280 mgfdL. to 320 mg/dL. The most appropricte management is a) Add metiormin ') Change to ancther type of insulin ©) Increase his caloric intake ©) Redistribute his caloric intake 6) Increase his insulin dose Question Explanation: ‘his paticnt has inadequately tweated insulin dependent diabetes. The goal serum glucose for therapy is < 120 mg/dL. Since there is no indication that the insulin type is incorrect (Lypoglycemia), the patient simply requires an increased dosage of his current insulin type. Metformin is an oral antihyperalycemic agent that acts by increasing peripheral utlization of glucose. The primary purpose of ‘mecications such as this is to delay or avoid the requirement for exogenous insulin. Once a patient is on insulin, these drugs are of litle benefit. Secondarily, most pationts with non insulin dependent diabetes have relative "ixsulin resistance" and ite for this reason that glucose utilization drugs are employed as first line management. Tais patient's symptoms of increased weight loss and appetite are a finction of is poor glucose control, not ofinadequete calonc intake. Foorly controlled insulin dependent diabetics are often thn as they waste muscle to liberate glucose in an attempt to provide glucose for the body. The defect is with the inability to utilize the gucose already present co the end result ic hyperglycemia and muscle wasting, For sinsilar reacons, there is no need to alter his caloric distribution, 2124/2014 2:18:51 PM “Mark this question = Question Id : 64448 Question 11 of 30 A 35 year old laborer comes to the office because of 2 kg (5-Ib) weight loss and an increased appetite. He has diabetes melitus and has been taking insulin in divided doses. He says thathome monitoring of his serum glucose concentration has shows values from 280 mgfdL. to 320 mg/dL. The most appropricte management is a) Add metiormin ') Change to ancther type of insulin ©) Increase his caloric intake ©) Redistribute his caloric intake o © 2) Increase his insulin dose Question Explanation: ‘his paticnt has inadequately tweated insulin dependent diabetes. The goal serum glucose for therapy is < 120 mg/dL. Since there is no indication that the insulin type is incorrect (Lypoglycemia), the patient simply requires an increased dosage of his current insulin type. Metformin is an oral antihyperalycemic agent that acts by increasing peripheral utlization of glucose. The primary purpose of ‘mecications such as this is to delay or avoid the requirement for exogenous insulin. Once a patient is on insulin, these drugs are of litle benefit. Secondarily, most pationts with non insulin dependent diabetes have relative "ixsulin resistance" and ite for this reason that glucose utilization drugs are employed as first line management. Tais patient's symptoms of increased weight loss and appetite are a finction of is poor glucose control, not ofinadequete calonc intake. Foorly controlled insulin dependent diabetics are often thn as they waste muscle to liberate glucose in an attempt to provide glucose for the body. The defect is with the inability to utilize the gucose already present co the end result ic hyperglycemia and muscle wasting, For sinsilar reacons, there is no need to alter his caloric distribution, ‘Marke this question << => Question Td : $1431 Question 12 of 30 ‘Nephrolithiasis is a common mode of presentation of which of the following disorder? a) Hyperparathyroicism. ) hyporparathyroidism, ©) Crohn's disease. 9) Gout 8) Diabetes Question Explanation: Renal stones de to hypercalciuria are a common presentation of primary kyperparathyroicism The other conditions are not associated with the nephrolthiasis ‘Marke this question << => Question Td : $1431 Question 12 of 30 ‘Nephrolithiasis is a common mode of presentation of which of the following disorder? Y © a) Hyperparathyroidism. +b) hyporparathyroidism. c) Crohn's disease dD Gout. ) Diabetes Question Explanation: Renal stones de to hypercalciuria are a common presentation of primary kyperparathyroicism The other conditions are not associated with the nephrolthiasis 2/24/2014 2:19:17 PM ‘Mark this question e=> Question Id : 90729 Question 13 of 30 All of the following can result in hypothyroicism, EXCEPT a) Sarcoidosis b) Sheehan's syndrome c) Ovarian teratoma 4) Lithium, €) Hashimoto’s thyroiditis Question Explanation: Ectopic production of thyroid hormones caused by conditions such as an ovarian teratoma resub in thyrotoxicosis. Primary hypothyroidism accounts for more than 90% of the cases ofhypothyroidicm. Autoimmue destructive processes, such as Hashimoto ‘s thyroiditis or Grave's disease, evennually result in hypothyroidism, Drugs, such 2s lthium and sulfonamides, which interfere with thyroid hotmone biosynthesis, also resultin the condition. Secondary hypothyroidism results from deficient pituitary thyrotrophs. Conditions, such as neoplasms, surgery ot Sheehan's syndrome, which result in panhypopinutarism, cause hypothyroidism Hypethalamic hyp thyroidism, due to decreased TRH secretion, leads to decreased secretion of TSH. Conditions, such as sarceidosis, which may infitrare the hypothalamus, or infectious processes, may also lead to hypothyroidism. 2/24/2014 2:19:17 PM ‘Mark this question e=> Question Id : 90729 Question 13 of 30 ‘All ofthe following can result in hypothyroidism, EXCEPT a) Sarceidosis b) Sheehan’s syndrome JY © c) Ovarian teratoma 4) Lithia ©) Hashimoto's thyroicltis Question Explanation: Ectopic production of thyroid hormones caused by conditions such as an ovarian teratoma resub in thyrotoxicosis. Primary hypothyroidism accounts for more than 90% of the cases ofhypothyroidicm. Autoimmue destructive processes, such as Hashimoto ‘s thyroiditis or Grave's disease, evennually result in hypothyroidism, Drugs, such 2s lthium and sulfonamides, which interfere with thyroid hotmone biosynthesis, also resultin the condition. Secondary hypothyroidism results from deficient pituitary thyrotrophs. Conditions, such as neoplasms, surgery ot Sheehan's syndrome, which result in panhypopinutarism, cause hypothyroidism Hypethalamic hyp thyroidism, due to decreased TRH secretion, leads to decreased secretion of TSH. Conditions, such as sarceidosis, which may infitrare the hypothalamus, or infectious processes, may also lead to hypothyroidism. Mark this question & => Question Td : 103022 Question 14 of 30 All of the following predispose to calcium stone formation, EXCEPT 2) Hyperuricosuia ) Primary hyp oparathyroidism ©) Distal renal tubular acidosis & Jejnnal ileal bypass €) Idiopathic hypercalciuria Question Explanation: Primary hyperparatiyroidism results in hypercalciuria, which raises the urine super saturation of calcium phosphate andlor calc‘ oxalate. Hyperuricosuria probably induces stone formation by heterogeneous nucleation of calcium oxalate by crystals of sodium tyckogen urate or uric acid, Distal renal tabular acidosis prevents the establishment of a normal pH gradient between urine and blood The resuiting allcalotic urine, associeted with hypercalciuria and alow urine citrate level, cause a supersaturation with calcium phosphate, predisposing to calcium phosphate stone formation, Fat malabsorption caused by conditions such as jejunelileal bypass, ileal resection, or chronic diseases of the pancrees, results in overabsorption of dietary oxalate Hyperoxaluria leads to tubulointerstitial nephropathy and stone formation. Idiopathic bypercalciuriais the most frequent abnormality found in patients with a family history of uephrolihiasis and is associated with an increase in intestinal calcium absorption. Mark this question & => Question Td : 103022 Question 14 of 30 Al of the following predispose to calcinm stone formation, EXCEPT a) Hyperuricosuria Y © b) Primary hypoparathyroidism c) Distal renal tubular acidosis 4) Jejunal ileal bypass e) Idiopathic hypercalciuria Question Explanation: Primary hyperparatiyroidism results in hypercalciuria, which raises the urine super saturation of calcium phosphate andlor calc‘ oxalate. Hyperuricosuria probably induces stone formation by heterogeneous nucleation of calcium oxalate by crystals of sodium tyckogen urate or uric acid, Distal renal tabular acidosis prevents the establishment of a normal pH gradient between urine and blood The resuiting allcalotic urine, associeted with hypercalciuria and alow urine citrate level, cause a supersaturation with calcium phosphate, predisposing to calcium phosphate stone formation, Fat malabsorption caused by conditions such as jejunelileal bypass, ileal resection, or chronic diseases of the pancrees, results in overabsorption of dietary oxalate Hyperoxaluria leads to tubulointerstitial nephropathy and stone formation. Idiopathic bypercalciuriais the most frequent abnormality found in patients with a family history of uephrolihiasis and is associated with an increase in intestinal calcium absorption. 22472014 2:49:47 PM ‘Mark this question & => Question 15 of 30 Euthyroid goiters are NOT associated with which of the fellowing? a) Endemic iodine deficiency ) Acute thyroiditis 2) Thyroid tumors. 4) Chronic lymphocytic thyroidtis ¢) Chortocarcinoma, Anowor (ERRATA) ner tcors Explanation Repos An Error Question Explanation: Question Td : 104102 Enthyroid goiters are not associated with choriocarcinoma, an invasive cancer originating in placental tissue, but can he seen in all of the other conditions 22472014 2:49:47 PM ‘Mark this question & => Question 15 of 30 Euthyroid goiters are NOT associated with which of the fellowing? a) Endemic iodine deficiency ) Acute thyroiditis 2) Thyroid tumors. 4) Chronic lymphocytic thyroidtis JY © 2) Choriocarcinoma. Anowor (SERRATE) ner cors Explanation Repos An Error Question Explanation: Question Td : 104102 Enthyroid goiters are not associated with choriocarcinoma, an invasive cancer originating in placental tissue, but can he seen in all of the other conditions ‘Mark this question & => Question Td : 121158 Question 16 of 30 ‘Which one of the fellowing syndrome is manifested by elevated plasma PTH, hypocalcemia, hyperphosphatemtia, and peripheral resistance ta PTH action? a) Panhypopinitarism, +) Acute tubular necrosis ¢) Pseudohypoparathyroidism, 4) Primary hypoparathyroidism, 2) Secondary hyp oparathyroilsm. Question Explanation: Pseudotypoparathyroidism is manifested by elevated plasma parathyroid hormone levels, hypocalcemia, hyperphosphatemia, and peripheral resistance to parathyroid hormone action, Pathypopitutarism would be associated with low plasma parathyroid levels and no peripheral resistance. Acute tubular necrosis does not cause this constellation of findings Primary and secondary hypoparathyroidism are associated with low plasma levels of PTH and no periphoral resistance to PTH. ‘Mark this question & => Question Td : 121158 Question 16 of 30 ‘Which one of the fellowing syndrome is manifested by elevated plasma PTH, hypocalcemia, hyperphosphatemtia, and peripheral resistance ta PTH action? a) Panhypopinitarism, +) Acute tubular necrosis Y © 0) Pseudohypoparathyroidism. 4) Primary hypoparathyroidism, 2) Secondary hyp oparathyroilsm. Question Explanation: Pseudotypoparathyroidism is manifested by elevated plasma parathyroid hormone levels, hypocalcemia, hyperphosphatemia, and peripheral resistance to parathyroid hormone action, Pathypopitutarism would be associated with low plasma parathyroid levels and no peripheral resistance. Acute tubular necrosis does not cause this constellation of findings Primary and secondary hypoparathyroidism are associated with low plasma levels of PTH and no periphoral resistance to PTH. ‘Mark this question & => Question Td : 136492 Question 17 of 30 All ofthe following are features of kypocaloemia, EXCEPT a) Tetany ») Seizures ¢) First degree heart block 4) Bronchospasm 6) Paresthesias Question Explanation: First degree heart block is not associated with hypocalcemia The electrocardiographic finding that is associated with hypocalcemia is, a prolonged QT interval. Tetany, seizures, bronchospasm, and paresthesias all are manifestations of hypocalcemia ‘Mark this question & => Question Td : 136492 Question 17 of 30 All of the following are features of hypocalcemia, EXCEPT a) Tetany b) Seimres Y © 0) First degree heart block, d) Bronchospasm ©) Paresthesias Question Explanation: First degree heart block is not associated with hypocalcemia The electrocardiographic finding that is associated with hypocalcemia is, a prolonged QT interval. Tetany, seizures, bronchospasm, and paresthesias all are manifestations of hypocalcemia 2i24/2014 2:50:26 PM Mark this question & => Question Td : 139010 Question 18 of 30 A. 36 year old woman with a painless palpable neck mass undergoes hemithyroidectomy. The bistopathology shows a single large nodule occupying one pole, which on frozen section shows benign colloid follicles. The nodule is delimited by bands of fibrous tissue. ‘The most likely dhagnosis is ) Diffuse nontoxic goiter ) Multinodular goiter ©) Subacute thyroiditis 4) Thyroid adenoma 8) Thyroid cyst Question Explanation: ‘Thyroid (follicular) adenoma is a common beniga lesion that is often surgically resected because of concem that itmay be cancer. The lesion consists of a single large nodule of thyroid tissue (ap to 5 cm diameter) the histology of which i often similar to that af the normal thyroid gland. Patients are generally euthyroid, Diffuse nontoxic goiter and multinodular goiter are stages of the same process, ‘with multinodular goiter producing a large mass with a more nodular texture Subacute thyroiditis causes painful thyroid enlargement. A thyroid cyst s a common beniga cystic space that develops in some thyroids 2i24/2014 2:50:26 PM Mark this question & => Question Td : 139010 Question 18 of 30 A356 year eld woman with a painless palpable neck mats undergoes hemithyroidectemy. The kistopathology shows a single large nodule occupying one pele, which on frozen section shows benign colloid follicles. The nodule is delimited by bands of ftrous tissue. ‘The most ikely dagnosis is 2) Diffuse nontoxic geiter 6) Muttinodular goiter ©) Subacute thyroiditis Y © & Thyroid adenoma 6) Thyroid cyst Question Explanation: ‘Thyroid (follicular) adenoma is a common beniga lesion that is often surgically resected because of concem that itmay be cancer. The lesion consists of a single large nodule of thyroid tissue (ap to 5 cm diameter) the histology of which i often similar to that af the normal thyroid gland. Patients are generally euthyroid, Diffuse nontoxic goiter and multinodular goiter are stages of the same process, ‘with multinodular goiter producing a large mass with a more nodular texture Subacute thyroiditis causes painful thyroid enlargement. A thyroid cyst s a common beniga cystic space that develops in some thyroids 2242014 2:50:43 PM ‘Mark this question & => Question Td : 162344 Question 19 of 30 A 63 year old woman has hemoglobin Alc of 8.9%, and it is now recommended that she begins medication for diabetes. She takes Enalapril and aspirin and has no allergies. She also has congestive heart failure, hypertension, and hypercholesterolemia Blood tests reveal a creatinine of 17 mg/dL. The appropriate medication to start at this time is, a) Glipizide ) Insulin ©) Metformin ©) Meiformin and Glipizide e) Rosiglitazone Question Explanation: Tn the inital decision about the choice of crug to manage hyperglycemia, one must take into account the comorbidities of the patient and relative contraindications, Insulin secretagogues such as sulfonyiureas, alpha-ghcosidase inhibitors, thiazolidinediones (Cglitazones"), biguanides, and insulin are all approved for monotherapy for diabetes. Most paticats are initially started on either a sulfonylurea or metformin, and insulin is usually given after oral agents have failed, Ttis important to note thet sulfonylureas are contraindicated for those with significant hepatic and liver dysfunction. Glipizide primarily undergoes hepatic clearance and should be used preferentially in patients with mid-to moderate renal dysfunction Meiforman, a biguaride, has a serious potential complication of lactic acidosis, The risk of lactic acidosis is increased in patients with heart falure, liver disease, severe hypoxia, any form of acidosis, intravenous contrast administration, and renal insufficiency, itis recommended that use of metformin be avoided in men with creatinine higher than 1.5 ma/dL and women with creatinine higher than 14 mg/dL. Thiazclidinediones such as rosiglitazone reduce insulin resistance by binding PPAR gamma receptors, and older generations ofthis clase of drugs are associated with liver tonicity. They are associated with exacerbations of congestive heart failure and peripheral edema. 2242014 2:50:43 PM ‘Mark this question & => Question Td : 162344 Question 19 of 30 A 63 year old woman has hemoglobin Alc of 8.9%, and it is now recommended that she begins medication for diabetes. She takes Enalapril and aspirin and has no allergies. She also has congestive heart failure, hypertension, and hypercholesterolemia Blood tests reveal a creatinine of 17 mg/dL. The appropriate medication to start at this time is, Y © @) Glipizide ) Insulin ©) Metformin ©) Meiformin and Glipizide e) Rosiglitazone Question Explanation: Tn the inital decision about the choice of crug to manage hyperglycemia, one must take into account the comorbidities of the patient and relative contraindications, Insulin secretagogues such as sulfonyiureas, alpha-ghcosidase inhibitors, thiazolidinediones (Cglitazones"), biguanides, and insulin are all approved for monotherapy for diabetes. Most paticats are initially started on either a sulfonylurea or metformin, and insulin is usually given after oral agents have failed, Ttis important to note thet sulfonylureas are contraindicated for those with significant hepatic and liver dysfunction. Glipizide primarily undergoes hepatic clearance and should be used preferentially in patients with mid-to moderate renal dysfunction Meiforman, a biguaride, has a serious potential complication of lactic acidosis, The risk of lactic acidosis is increased in patients with heart falure, liver disease, severe hypoxia, any form of acidosis, intravenous contrast administration, and renal insufficiency, itis recommended that use of metformin be avoided in men with creatinine higher than 1.5 ma/dL and women with creatinine higher than 14 mg/dL. Thiazclidinediones such as rosiglitazone reduce insulin resistance by binding PPAR gamma receptors, and older generations ofthis clase of drugs are associated with liver tonicity. They are associated with exacerbations of congestive heart failure and peripheral edema. 2/24/2014 2:50:58 PM “Mark this question => Question Id : 178167 Question 20 of 30 A 32 year old men presents has complaint of inabiity to impregnate his wife. He wants to find out ifhe is infertile. He deries fatigue and states that he has 2 decreased libido, On exam, his testes appear normal. Lab testing would likely reveal that in additon to alow spenm count he has elevated level of which hormone? a) Folicle-stinmulating hormone (FSH) ') Luteinizing hormone (LH) ©) Luteinizing hormone-releasing hormone (LHRE) 4) Prolactin 8) Testosterone Question Explanation: Prolactin receptors are located on Leydig cells in the testes. Under normal conditions, prolactin synergizes with LET to stimulate testosterone production by Leydig cells. In males with hyperprolactinemia secondary to a pitutary tumor, elevated prolactin levels interfere with testosterone production, Excessive prolactin reduces the number of LH receptors and/or inhibits incracelular events stimulated by LH. Hyperprolactinemia may also decrease the pulsatile nature of LH secretion, also cansing a decrease in testosterone production, Testosterone production is required for spermatogenesis, so elevated prolactin decreases sperin count and frequently causes infertility. An increase in FSH, LH, and LHRH would increase testosterone production rather than decrease it, LHRH ss released from the hypothalamus and acts on the antesior pituitary to stimulate LH and FSH release, LH acts on the Leydig cells of the testes to increase testosterone production which acts on the Sertoli cells to fasilzate spermatogenesis. An increase in testosterone ‘would result in an increase in spermatogenesis and would not result in infertility. 2/24/2014 2:50:58 PM “Mark this question => Question Id : 178167 Question 20 of 30 A. 32 year old man presents has complaint of inabiity to impregnate his wife, He wants to find outifhe is infertile. He denies fatigue and states thathe has 2 decreased libido. On exam, his testes appear normal. Lab testing would likely reveal that in addition to alow penn count he hac elevated level of which hormone? 2) Folicle-stinulating hormone (FSH) +) Luteinizing hormone (LH) ) Luteinizing hormene-releasing hormone (LHRH) Y @ & Prolactin 6) Testosterone Question Explanation: Prolactin receptors are located on Leydig cells in the testes. Under normal conditions, prolactin synergizes with LET to stimulate testosterone production by Leydig cells. In males with hyperprolactinemia secondary to a pitutary tumor, elevated prolactin levels interfere with testosterone production, Excessive prolactin reduces the number of LH receptors and/or inhibits incracelular events stimulated by LH. Hyperprolactinemia may also decrease the pulsatile nature of LH secretion, also cansing a decrease in testosterone production, Testosterone production is required for spermatogenesis, so elevated prolactin decreases sperin count and frequently causes infertility. An increase in FSH, LH, and LHRH would increase testosterone production rather than decrease it, LHRH ss released from the hypothalamus and acts on the antesior pituitary to stimulate LH and FSH release, LH acts on the Leydig cells of the testes to increase testosterone production which acts on the Sertoli cells to fasilzate spermatogenesis. An increase in testosterone ‘would result in an increase in spermatogenesis and would not result in infertility. 2242014 2:51:13 PM “Mark this question => Question Id : 194600 Question 21 of 30 A5/ year old man is diagnosed with acromegaly. He has normal vsual fields. Which one of the following is the most appropriate treatment for this patient? a) Bromocriptine } Cabergoiine ©) Radiotherapy 4) Somatostatin anclogue therapy ¢) Trans-sphenoidal hypophysectomy Avewor [UEQVIRNBNEN) OtorUsore Explanation Report An Eiror Question Explanation: ‘The most appropriate treatment for acromegaly in this middle aged man which may prove curative is surgery. Somatostatin therapy, although frequently effective in reducing GH, would not be advocated in this young patient who would require liflong therapy. 2242014 2:51:13 PM “Mark this question => Question Id : 194600 Question 21 of 30 ADI year old man is ckagnosed with acromegaly. He has normal visual fields. Which one of the following is the most appropriate treatment for this patient? a) Bromocriptine } Cabergoiine ) Radiotherapy 4) Somatostatin analogue therapy Y © c) Trans-sphenoidal hypophysectomy: Avewor [UEIPIRNBHEN) OtorUsore Explanation Report An Error Question Explanation: ‘The most appropriate treatment for acromegaly in this middle aged man which may prove curative is surgery. Somatostatin therapy, although frequently effective in reducing GH, would not be advocated in this young patient who would require liflong therapy. Mark this question ez Question Id : 195115 Question 22 of 30 A 38 year old female presents with polyuria and is passing 4 liters of urine per day. She was recently started on anew medication. Her serum soduum is 144 mmol/L (137-144), plasmna osmolality is 299mosmoVL (275-290) and urine osmolality is 210mosmol/L. (350-1000), The drug that was lkely prescribed is a) Aspirin 1b) Fluoxetine ©) Glibencamide ) Lithium ©) Metoprolol Question Explanation: This lady has eunatraemia, kypertonicity thigh serum osmolality) inappropriately dilute urine) which is consistent with Diabetes insipidus. Of the drugs listed Lathiam would be the most likely cause a nephrogenic DL Mark this question ez Question Id : 195115 Question 22 of 30 A 38 year old female presents with polywia and is passing 4 liters of urine per day. She was recently started on anew medication. Her serum sockum is 144 mmol/L (137-144), plasma osmolality is 29SmosmolL (275-290) and urine osmolality is 210mosmoVL. (350-1000). The drug that was likely prescribed is a) Aspirin 'b) Fluoxetine c) Glibenclamide YO A Lithinm e) Metoprolol Question Explanation: This lady has eunatraemia, kypertonicity thigh serum osmolality) inappropriately dilute urine) which is consistent with Diabetes insipidus. Of the drugs listed Lathiam would be the most likely cause a nephrogenic DL ‘Mark this question = => Question Id : 195196 Question 23 of 30 A 19 year old boy is concemed regarding his pubertal development hair. Both testes are approximately 5 mls in volume (Normal Sms), No other atmorrality ie present. Labs show LH level of 3 3 mol/L (3-10), FSH level of 5.5 millML. (3-10) and testosterone level of 5.5 mol. (9-30). The likely diagnosis is a) Anorexia nervosa ) Craniopharyngioma c) Kalman seérome 4) Blinefeter’s testicular faiure ) Primary testicular fedure Answer | Besianation Other User's Explanation Report An Error Question Explanation: This young male has delayed puberty with hypogonadetrophic hypogonadism, The most lkkely explanation would be Kallnann cyndrome, Elinefelters would be aczociated with elevated LEVESH as would primary testicular failure. His height of 1.2m cuggesting that he is tal would argue against this being anorexia despite his thin appearance as if anorexia had preceded puberty then his stature should also have been affected, ‘The only other possbiity is Cranopharyngioma but ths is probably less lcely in an otherwise well man and other possble features should be included to suggest this diagnosis visual fields maybe. You may wish to cry foul at this point and cay that anosmia chould be given ac a clue to Kalknenns. However, in theory how often do you adle about sence of smell lt alone vest or it! ‘Mark this question = => Question Id : 195196 Question 23 of 30 A.19 year old bor is concemed regarding his pubertal development hair. Both testes are approximately 5 ms in volame (Normal 151rls) No other abnormality is present. Labs show LH level of 3.3 mUI/L (3-10), FSH level of 5.5 mUJ/L (3-10) and testosterone level of 5.5 nmol/L (9-30). The likely diagnosis is a) Anoresia nervosa b) Craniopharyngioma ¥ © ¢) Kalimann syndrome d) Klinefelter‘s testicular failure ) Primary testicular failure Answer | Besianation Other User's Explanation Report An Error Question Explanation: This young male has delayed puberty with hypogonadetrophic hypogonadism, The most lkkely explanation would be Kallnann cyndrome, Elinefelters would be aczociated with elevated LEVESH as would primary testicular failure. His height of 1.2m cuggesting that he is tal would argue against this being anorexia despite his thin appearance as if anorexia had preceded puberty then his stature should also have been affected, ‘The only other possbiity is Cranopharyngioma but ths is probably less lcely in an otherwise well man and other possble features should be included to suggest this diagnosis visual fields maybe. You may wish to cry foul at this point and cay that anosmia chould be given ac a clue to Kalknenns. However, in theory how often do you adle about sence of smell lt alone vest or it! ‘Mark this question & => Question Id : 195617 Question 24 of 30 A32 year old female with a one year history of galactorthoea and amenorrhoea informs you that she does not want to become pregnant. On exam: there is galactorrhoea, Labs confirm the dlagorie ofa macroprolactinoma MRI reveals a 1.5 om pituitary tumor. Appropriaic treatment for this woman is a) Cabergoline therapy ) Combined oral contraceptive ©) Pituitary surgery 4) Somatostatin analogue therapy ¢) Stereotactic pituitery irradiation Question Explanation: This young woman has a macroprolactinoma and these are exqnisitely sensitive to dopamine agonist therapy and rapid tumor reduction with restoration of menses and cessation of galactorrhoca expected. Piuéary surgery is rarely required in prolactinomas and is generally reserved for patents intolerant or resistant to dopamine agonist therapy Even with large tamors that compress the chiasm, these can treated with dopamine agonists with rapid reduction in size and relief of pressure. The fact that she does not want to become pregnant is a bit of an irrelevance. One would sill treat her with dopamine agonist therapy and suggest use of appropriate contraception which could include the OCP, ‘Mark this question & => Question Id : 195617 Question 24 of 30 A. 32 year old female with a one year history of galactorthoea and amenorrhoea informs you that she does not want to become pregnant. On exam there is galactorrhoea. Labs confirm the diagnosis of a macroprolactinoma MRI reveals a 1.5 cm pituitary tumor. Appropriate treatment for this woman is Y © a) Cabergoline therapy 'b) Combined oral contraceptive c) Pituitary surgery d) Somatostatin analogue therapy ©) Stereotactic pituitary irradiation Question Explanation: This young woman has a macroprolactinoma and these are exqnisitely sensitive to dopamine agonist therapy and rapid tumor reduction with restoration of menses and cessation of galactorrhoca expected. Piuéary surgery is rarely required in prolactinomas and is generally reserved for patents intolerant or resistant to dopamine agonist therapy Even with large tamors that compress the chiasm, these can treated with dopamine agonists with rapid reduction in size and relief of pressure. The fact that she does not want to become pregnant is a bit of an irrelevance. One would sill treat her with dopamine agonist therapy and suggest use of appropriate contraception which could include the OCP, 2124/2014 2:52:10 PM ‘Mark this question & => Question Td : 195701 Question 25 of 30 A.44 year old manis concemed about his risk of developing diabetes. His mother and maternal uncle both have dicbetes. He has central oberity with a waist measurement of 110 cm. BP ic 130/22 mmHg and BMI ic 30.2 kg/m2. Fasting cholesterol is 5.2 mmol/L (5.2), wiglycerides are 1.4 mmol/L (0.45-1.69), EDL chelesterolis 1.1 mmol/L (1.55) and fasting glucose is 6.2 mmol/L (3.0- 6.0), His observation that fills the criteria for chagnosis of the metabolic syndrome is 8) Blood pressure of 130/82 mamElg ») BMI of 30.2 kgin2 ¢) Fasting plasma gucoce of 6.2 mel 4) HDL concentration of 1.1 mmol ¢) Triglyceride concentration of 14 mamol Anower [UBQRSEREN) incr User's Explanation Report An or Question Explanation: ‘The metabolic syncrome is becoming hugely important as a cluster of features associated with increased cardiovascular and diabetes, tisk, The condition is defined by various criceria the latest of which is global definition for the IDF as central obesity (>=94em for inea, >=8 cm for women) plus any two of the following © Hypertiglyceridaemia >17 mmol! + Low HDL concentration <1.03 mmol male, «1.29 mmol female © BP > of = 130/85 mmHg, or on treatment for hypertension Fasting glucose > or =5.6 mmol/l, ot known to have type 2 diabetes. Thus. in our patient's case the elevated fasting glucose of 6.3 mmol fills this diggnostic criterion The BMTis not a function of the diagnostic criteria as the waict circumference appears to be a far more powerful predictor of risk, 2124/2014 2:52:10 PM ‘Mark this question & => Question Td : 195701 Question 25 of 30 44 year old man is concemed about his tisk of developing diabetes. His mother and maternal uncle both have diebetes. He has central oberity with a waist measurement of 110 cm. BP ic 130/22 mmHg and BMI ic 30.2 kg/m2. Fasting cholesterol is 5.2 mmol/L (5.2), triglycerides are 1.4 mmoVL (0.45-1.69), HDL cholesterolis 1.1 mmo¥L (1.55) and fasting glucose is 6.2 mmol/L (3.0- 6.0), His observation that fills the criteria for chagnosis of the metabolic syndrome is 8) Blood pressure of 130/82 mamElg ») BMI of 30.2 kgin2 V © c) Fasting plasma glucose of 6.2 mmol/l 4) HDL concentration of 1.1 mmol ¢) Triglyceride concentration of 14 mamol Anower [UBQRSEREN) incr User's Explanation Report An or Question Explanation: ‘The metabolic syncrome is becoming hugely important as a cluster of features associated with increased cardiovascular and diabetes, tisk, The condition is defined by various criceria the latest of which is global definition for the IDF as central obesity (>=94em for inea, >=8 cm for women) plus any two of the following © Hypertiglyceridaemia >17 mmol! + Low HDL concentration <1.03 mmol male, «1.29 mmol female © BP > of = 130/85 mmHg, or on treatment for hypertension Fasting glucose > or =5.6 mmol/l, ot known to have type 2 diabetes. Thus. in our patient's case the elevated fasting glucose of 6.3 mmol fills this diggnostic criterion The BMTis not a function of the diagnostic criteria as the waict circumference appears to be a far more powerful predictor of risk, ‘Mark this question << => Question Td : 195762 Question 26 of 30, AAR year old diabetic male managed with metformin 1 g twice daily has recently been started on exanatide because he is morbidly obese and stil failing to achieve adequate blood glucose control. His BMI is 41. HbAlcis 7.2% (<5.5). He asks about blood glucose monitor for his glucoses. Which statement is consistent with the recommendations in the consensus? a) He should no need to self monitor his ghcoses b) He should monitor fasting glucoses three times per week ©) He should monitor fasting alucoses every day 4) He should monitor morning fasting an pre-prandial alucoses three times per weelc ©) He should monitor morning fasting an post-prandiel gucoses three times per weele Anewor [UEIENNT) ter veer Explanation Report An Eo Question Explanation: Generally, if patients are not using insulin, sulphonylureas or ginides (Repaglinide or Netaglinide), then the consensus does not recommend self-monitoring of blood glucose levels. With respect to insulin titration however, self monitored blood glucose sesults play a crucial partin appropriate dose adjustinent. The consensus recommends a target of between 3.9 and 7.2 mmol for fasting and pre-prandial glucose levels. IFfasting levels are in range yet the Hb.AL1c is elevated, post-prandial monitoring is recommended, aiming for glucose levels of less then 10 mmol ‘Mark this question << => Question Td : 195762 Question 26 of 30, A.48 year old diabetic male managed with metfermin 1 g twice daily has recently heen started on exanatide hecause he ie morbidly obese and stil filing to achieve adequate blood glucose control. His BMI is 41. HbAlc is 7.2% (<5.5). He asks about blood glucose monitor for his glucoses. Which statement is corsistent with the recommendations in the consensus? Y © a)He chouldno need to self monitor his ghicoses 'b) He should montor fasting glucoses three tines per week ©) He should monitor fasting glusosec every day 4) He should montor morning fasting an pre-prandial glacoses three times per week ©) He should monitor morning fasting an post-prandial gucoses three times per welt Anewor [UEIENNT) ter veer Explanation Report An Eo Question Explanation: Generally, if patients are not using insulin, sulphonylureas or ginides (Repaglinide or Netaglinide), then the consensus does not recommend self-monitoring of blood glucose levels. With respect to insulin titration however, self monitored blood glucose sesults play a crucial partin appropriate dose adjustinent. The consensus recommends a target of between 3.9 and 7.2 mmol for fasting and pre-prandial glucose levels. IFfasting levels are in range yet the Hb.AL1c is elevated, post-prandial monitoring is recommended, aiming for glucose levels of less then 10 mmol 2242014 2:52:34 PM Mark this question & => Question Id : 196181 Question 27 of 30 A.44 year old woman presents with excessive hair growth on her face, chest and lower abdomen. Which one may be associated with her condition? a) Cyproterone ) Hypoadcenalisin ©) Minoxidl & Moxonidine 6) Valproate newer (UBENERREN) oer users Explanation Report An Error Question Explanation: Drugs causing hirsutiom/hypertrichosic include minoxidil (not moxonidine -a centrally acting antihypertensive), phenytoin (not valproate) and cyclosporin. Polycystic ovaries and congenital adrenal hyperplasia are associated with increased androgens and hirsutism Hypoadrenalism may tbe associated with loss of hair especially Pubic hair ‘Treatment of hirsutism is with anti-androgens (cyproterone, spironolactone), reduction of fice androgens (oestrogen therapy, OCP, weight loss), and cosmetic treatment for removal of bait 2242014 2:52:34 PM Mark this question & => Question Id : 196181 Question 27 of 30 A.44 year old woman presents with excessive hair growth on her face, chest and lower abdomen. Which one may be associated with her condition? a) Cyproterone ) Hypoadcenalisin Y © 0) Minoxidil & Moxonidine 6) Valproate newer (UBENERREN) oer users Explanation Report An Error Question Explanation: Drugs causing hirsutiom/hypertrichosic include minoxidil (not moxonidine -a centrally acting antihypertensive), phenytoin (not valproate) and cyclosporin. Polycystic ovaries and congenital adrenal hyperplasia are associated with increased androgens and hirsutism Hypoadrenalism may tbe associated with loss of hair especially Pubic hair ‘Treatment of hirsutism is with anti-androgens (cyproterone, spironolactone), reduction of fice androgens (oestrogen therapy, OCP, weight loss), and cosmetic treatment for removal of bait ‘Mark this question = => Question Td : 196507 Question 28 of 30 A(51 year ole man presents with milley cischarge from his ripples. He has a history of depression and gastro-oesophageal reflux discase and is taking many medications. Plasma prolactin level is 650 mU/L (+ 360). The likely cause of his symptoms is a) Amitryptiline b) Cimetidine c) Fluoxetine 4d) Metoclopramnide ©) Omeprazole Answer | Bolanation Other User's Explanation Report An Error Question Explanation: This man has galactorrhoea and raised prolactin. The most likely culprit is metoclopramide throngh its action on dopamine. Tt releases prolactin through dopamine antagonism. ‘Mark this question = => Question Td : 196507 Question 28 of 30 A(51 year ole man presents with milley cischarge from his ripples. He has a history of depression and gastro-oesophageal reflux discase and is taking many medications. Plasma prolactin level is 650 mU/L (+ 360). The likely cause of his symptoms is a) Amitryptiline ) Cimetidine c) Fluoxetine Y © d) Metoclopramide ©) Omeprazole Answer | Bolanation Other User's Explanation Report An Error Question Explanation: This man has galactorrhoea and raised prolactin. The most likely culprit is metoclopramide throngh its action on dopamine. Tt releases prolactin through dopamine antagonism. ‘Mark this question & => Question Td : 196730 Question 29 of 30 ASB year old male's diabetes was deteriorating with blood glucose readings of 9-12 at home despite following a diet and taking ae taking Rosightazone for diabetes. His BMI is 29, BP is 128/74 melg and pulse is 63/min, He has some pining oedema in the lower limbs. He is obese with no organomegaly HbAlc is 8.5%. The best way to treat his elycaemis control is a) Add Gliclaside BOmg bd b) Increase Rosigttazone to 8mg daily c) Stop metformin and use Avandamet (combination of Metformin and Rosiglitazone) ) Subsitute Rosiglitazone with Gilaride 80mg bd ¢) Substinte Rosiglitazone with Pioglitazone mg daily Question Explanation: This patient is likely to be insulin resistant, however there is evidence of heart failure and fuid overload, so use of thiazolinediones Rosiglitazone or Fioglitazone) are absolutely contraindicated. These drugs promote fhid retention by means of an action on the collecting éucte of the kidney so promoting socium and water retention, Tae orly appropriate action thereffre is to stop Rosiglitazone, substituting this with the insulin secretagogue glclazide which will hopefilly improve his glycaemic contrcl ‘Mark this question & => Question Td : 196730 Question 29 of 30 ASB year old male's diabetes was deteriorating with blood glucose readings of 9-12 at home despite following a diet and taking ae taking Rosightazone for diabetes. His BMI is 29, BP is 128/74 melg and pulse is 63/min, He has some pining oedema in the lower limbs. He is obese with no organomegaly HbAlc is 8.5%. The best way to treat his elycaemis control is a) Add Gliclaside BOmg bd b) Increase Rosigttazone to 8mg daily c) Stop metformin and use Avandamet (combination of Metformin and Rosiglitazone) Y © & Substinite Rosiglitazone with Gilaride 80mg bd ¢) Substinte Rosiglitazone with Pioglitazone mg daily Question Explanation: This patient is likely to be insulin resistant, however there is evidence of heart failure and fuid overload, so use of thiazolinediones Rosiglitazone or Fioglitazone) are absolutely contraindicated. These drugs promote fhid retention by means of an action on the collecting éucte of the kidney so promoting socium and water retention, Tae orly appropriate action thereffre is to stop Rosiglitazone, substituting this with the insulin secretagogue glclazide which will hopefilly improve his glycaemic contrcl ‘Mark this question & Question Td : 216889 Question 30 of 30 A. S0-year-old woman with diabetes since childhood brings her glucose log shown as follows: [Sunday|/Monday|[Tuesiay|| Wednesday [Thursday|Friday [Saturday] |BAM |]101 122 145 KE 123 100 [90 Ls 2M 189 170 190 211 169 202 |150, em |fis4 fico fiz |fas as |sz_ pe PPM | 278 |[too 103 132 l11 108/100 Numbers reflect ghacose in mg/L, BAM sugars are fasting ‘Which of the following is the most appropriate management at this time? a) Begin a trial of oral medications 'b) Increase pre-breakfast regular insulin c) Increase pre-breakfast NPH insulin, d) Increase pre-dinner regular insulin €) Make no chenges in her current treatment Question Explanation: ‘This patient has farly good glucose control with the excepton of her 11 AM glucose measurement. These pre-lunch sugars rellect her morning regular insulin dose, AM regular insulin will peak in 2 to 4 hours, which should coincide with the increased serum glucose thet cocurs alter eating and digesting brealfact. Increasing AM regular inouin would therefore be helpfilin decreasing her 11 AM blood sugars Patients with type I dabetes do not respond wel to oral medications. This is because the mechanism for their poor ghucose control is different from that found in type TI dichetes. Type Tpatients have no problem with insulin resistance and no ability to increase their ovin insulin production; therefore, conventional oral diabetes medication wil not be effective. Because our patient's 4 pm sugars are fairly well controlled, adjusting her pre-brealfast NPH is incorrect. The 4 PM glucose measurements are a reflection of the patient's morring NPH insulin dose. Remembering that the peak action of NPH insulins 6 to 12 houre after adminictration chould help one remember that the NEHin the morning will help the patient control the glucose surge that occurs after imichtine. We risk hypoglycemia by increasing the NPE at this time. Increasing pre-dinner regular insulin is incorrect. Our patient's 9 PM sugars are a reflection of pre-dinner regular insulin doses. Although our patient does have elevated ghicose on Sunday. the remainder of her blood sugars at 9 PM is excellent. This isolated hyperglycemic episode might be caused by a dictary indiscretion on Sunday night. Further history would likely clarity ‘Making no changes at this time is not appropriate because our patient is a young woman with long history of diabetes. Good glucose control inthis patient may prevent or delay the complications of diabetes, including blindness end kidney disease. Our goal should be ta mimin the hload encare af a natient not effected hy diaheter ‘Mark this question & Question Td : 216889 Question 30 of 30 A. S0-year-old woman with diabetes since childhood brings her glucose log shown as follows: [Sunday|/Monday|[Tuesiay|| Wednesday [Thursday|Friday [Saturday] |BAM |]101 122 145 KE 123 100 [90 Ls 2M 189 170 190 211 169 202 |150, em |fis4 fico fiz |fas as |sz_ pe PPM | 278 |[too 103 132 l11 108/100 Numbers reflect ghacose in mg/L, BAM sugars are fasting Which of the following is the most appropriate management at this time? a) Begin a trial of oral medications J © b) Increase pre-breakfast regular insulin c) Increase pre-breakfast NPH insulin 4) Increase pre-dinner regular insulin ¢) Make no changes in her current treatment Question Explanation: ‘This patient has farly good glucose control with the excepton of her 11 AM glucose measurement. These pre-lunch sugars rellect her morning regular insulin dose, AM regular insulin will peak in 2 to 4 hours, which should coincide with the increased serum glucose thet cocurs alter eating and digesting brealfact. Increasing AM regular inouin would therefore be helpfilin decreasing her 11 AM blood sugars Patients with type I dabetes do not respond wel to oral medications. This is because the mechanism for their poor ghucose control is different from that found in type TI dichetes. Type Tpatients have no problem with insulin resistance and no ability to increase their ovin insulin production; therefore, conventional oral diabetes medication wil not be effective. Because our patient's 4 pm sugars are fairly well controlled, adjusting her pre-brealfast NPH is incorrect. The 4 PM glucose measurements are a reflection of the patient's morring NPH insulin dose. Remembering that the peak action of NPH insulins 6 to 12 houre after adminictration chould help one remember that the NEHin the morning will help the patient control the glucose surge that occurs after imichtine. We risk hypoglycemia by increasing the NPE at this time. Increasing pre-dinner regular insulin is incorrect. Our patient's 9 PM sugars are a reflection of pre-dinner regular insulin doses. Although our patient does have elevated ghicose on Sunday. the remainder of her blood sugars at 9 PM is excellent. This isolated hyperglycemic episode might be caused by a dictary indiscretion on Sunday night. Further history would likely clarity ‘Making no changes at this time is not appropriate because our patient is a young woman with long history of diabetes. Good glucose control inthis patient may prevent or delay the complications of diabetes, including blindness end kidney disease. Our goal should be ta mimin the hload encare af a natient not effected hy diaheter Mark this question => Question Td : 22303 Question 1 of 30 Aman aged 72 has hadi an indolent, unhealing ulcer at the heel of the right foot for several weeks. The ulcer is 3.5 cmin diameter, paitless, ts base locks dicty, and there is hardly any granulation tissue. The skin around the ulcer looks normal. The patient has no sensation of pin prick anywhere in that foot. Peripheral pulses are weal but palpable. He is obese, has varicose veins and poorly controlled type 2 diabetes melltus. Which of the following most accurately charactztizes the weer? 2) Ischemic ulcer due to arteriosclerosis ') Ischemic ulcer due to embolization ©) Diabetic ulcer due to trauma, neuropathy, and microvascular disease 6) Neoplastic in navure, probably squamous cell carcinoma 6) Stasis ulcer due to venous insufficiency Question Explanation: Diabetic ulcers typically develop at pressure points, and the heel is a favorite location The patient has evidence of neuropathy. and the correlation with the trauma inficted by the new shoes is classic. Ischemic ulcers, whether due to arteriosclerosis or embolization. ae typically seen atthe tip of the toes, as far away fiom the heart as one can get. Mark this question => Question Td : 22303 Question 1 of 30 Aman aged 72 has had an indolent, unhealing ulcer at the heel of the right foot for several weeks. The ulcer is 3.5 cm in diameter, paisless, its base locks dcty, and there is hardly any granulation tissue, The skin around the ulcer looks normal. The patient has no sensation of pin prick anywhere in that foot. Peripheral pulses are weak but palpable He is obese, has varicose veins and poorly controlled type 2 diabetes mellitus. Which of the following most accurately characterizes the ulcer? 2) Ischemic ulcer due to arteriosclerosis ‘b) Ischemic ulcer due to embolization Y © ©) Diabetic ulcer due to trauma, neuropathy, and microvascular disease 4) Neoplastic in nature, probably squamous cell carcinoma €) Stasis ulcer due to venous insufficiency Question Explanation: Diabetic ulcers typically develop at pressure points, and the heel is a favorite location The patient has evidence of neuropathy. and the correlation with the trauma inficted by the new shoes is classic. Ischemic ulcers, whether due to arteriosclerosis or embolization. ae typically seen atthe tip of the toes, as far away fiom the heart as one can get. Mark this question = => Question Td : 50730 Question 2 of 30 Out of the following, which is associated with thyroid diseases? 2) Newrofibromatosis +) Vitiigo 6) Erythema nodosum 4) Pemphigus vulgaris 6) Isthyosis vulgaris Answer [FESSBIRNBN) other Users Explana Question Explanation: Vitiigo is a loss of skin melanocytes that causes areas of skin depigmentation of varying sizes. Cause is unknown, but the condition may be autoimmune, as up to 1/3 of petieats have evidence of other autoimmune discase. Vitligo affects 0.5 to 2% of the population. Etiology is unknown, but melanocytes are lacking in affected areas; some patients have antibodies to melanin. Up to 30% have other autoitnmune antibodies (to thyroglobulin, adrenal cells, and parietal cells) or clinical autoimmune endocrinopathies (Addison's, disease. diabetes mellias, pernicious anemia, and thyroid dysfunction), leading to speculation that vitiligo is an autoimmune disease However, the selationship is unclear and may be coincidental The strongest association is with hyperthyroidism (Graves’ disease) and hypothyroidism (Hashimoto's thyroidtis). n Report An Error Mark this question = => Question Td : 50730 Question 2 of 30 Out ofthe following, which is associated with thyroid diseases? 2) Neurofibromatosis Y © b) Vitiligo ©) Erythema nodosum, 4) Bemphigns vulgaris 2) Icthyosis vulgaris Answer [FESSBIRNBN) other Users Explana Question Explanation: Vitiigo is a loss of skin melanocytes that causes areas of skin depigmentation of varying sizes. Cause is unknown, but the condition may be autoimmune, as up to 1/3 of petieats have evidence of other autoimmune discase. Vitligo affects 0.5 to 2% of the population. Etiology is unknown, but melanocytes are lacking in affected areas; some patients have antibodies to melanin. Up to 30% have other autoitnmune antibodies (to thyroglobulin, adrenal cells, and parietal cells) or clinical autoimmune endocrinopathies (Addison's, disease. diabetes mellias, pernicious anemia, and thyroid dysfunction), leading to speculation that vitiligo is an autoimmune disease However, the selationship is unclear and may be coincidental The strongest association is with hyperthyroidism (Graves’ disease) and hypothyroidism (Hashimoto's thyroidtis). n Report An Error Mark this question & => Question Td : 67145 Question 3 of 30 A tran has a semen analysis done. It shows abnormal motlity and shape, and a sotal sperm court of 950,000 (sperm count should be greater than 2,000,000). What would you advise him about his futare fetilty? 2) Chance of fertlity is 10% 1b) Chance of ferlity is 50% ©) He could have a child with in-vitro fertiization (TVE) ) He is impotent Question Explanation: Abnormal morphology (shape) and motility can prevent the sperm from reaching the egg. The sperm need motility to be able to swim well and survive for a number of hours n the female reproductive tract, IFthey do meet, abnormal-looking sperm might be incapable cof fertiization, The motility of this man’s spemm ie. poor and therefore he is unlikely to be eble to be fertie. In additon, infertley specialists have stated thet 1 million motile sperm is the mininum amount of spenm associated with a reasonable chance of preanancy success at intrauterine inserninetion However in-vitro fertilization IVF can be usedto treat Infertility dae to his olgospenniia. The procedure would involve controlled ovarian hyperstimulation, oocyte retrieval, and fertilization with, sperm, embryo culture, and embryo transfer. Impotence is defined as an inzbilty to achieve and/or maintain an erection. n Report An Error Mark this question & => Question Td : 67145 Question 3 of 30 Arman has 2 semen analysis done, It shows abnormal motlity and shape, and a total spetm count of 950,000 (sperm count should be greater than 2,000,000). What would you advise him about his futare fertility? 8) Chance of frtlity is 10% bb) Chance of ferity is 50% JV © o) He could have a child with in-vitro fertilization (VE) 6) Heis impotent Question Explanation: Abnormal morphology (shape) and motility can prevent the sperm from reaching the egg. The sperm need motility to be able to swim well and survive for a number of hours n the female reproductive tract, IFthey do meet, abnormal-looking sperm might be incapable cof fertiization, The motility of this man’s spemm ie. poor and therefore he is unlikely to be eble to be fertie. In additon, infertley specialists have stated thet 1 million motile sperm is the mininum amount of spenm associated with a reasonable chance of preanancy success at intrauterine inserninetion However in-vitro fertilization IVF can be usedto treat Infertility dae to his olgospenniia. The procedure would involve controlled ovarian hyperstimulation, oocyte retrieval, and fertilization with, sperm, embryo culture, and embryo transfer. Impotence is defined as an inzbilty to achieve and/or maintain an erection. n Report An Error 2242014 2:54:48 PM ‘Mark this question & => Question Td : 67205 Question 4 of 30 A.56 year old woman, while being investigated for long stancing hypertension, is found to have the folowing serum laboratory test values: normal creatine , total protein, albumin, and globulin, increased calcium and alkaline phosphatase, and decreased phosphorus ‘These Sindings suggest the presence of which of the following? a) Carcinoma metastatic to bone b) Excessive dietary calcium intake ©) Mutiple myeloma & Parathyroid adenoma ©) Sarcoidosis Question Explanation: Primary hyperparathyroicism is usualy the result of a single benign adenoma, Its a cause of secondary hypertension. Elevated parathyroid hormone (PTH) levels in the setting of hypercalcemia establish the clagnosis of yperparathyroidism. A decreased serum phosphate level may be seen. Increased bone turnover may be reflected in elevated levels of markers of bone formation (alkaline phosphatase) and bone resorption (urinary pyridinoline). 2242014 2:54:48 PM ‘Mark this question & => Question Td : 67205 Question 4 of 30 A.56 year old woman, while being investigated for long stancing hypertension, is found to have the folowing serum laboratory test values: normal creatine . total protein, albumin, and globulin: increased calcium and alkaline phosphatase, and decreased phosphorus ‘These Sndings suggest the presence of which of the following? a) Carcinoma metastatic to bone +b) Excessive distary celcinm intake ©) Mutiple myeloma WM © & Parathyroid adenoma ©) Sarcoidosis Question Explanation: Primary hyperparathyroicism is usualy the result of a single benign adenoma, Its a cause of secondary hypertension. Elevated parathyroid hormone (PTH) levels in the setting of hypercalcemia establish the clagnosis of yperparathyroidism. A decreased serum phosphate level may be seen. Increased bone turnover may be reflected in elevated levels of markers of bone formation (alkaline phosphatase) and bone resorption (urinary pyridinoline). “Mark this question >= Question Id : 67305 Question 5 of 30 A.48 year old man has persistent hypertension despite taking metoprolol, Enalapril and Nifedipine, Mo history palpitations or fushing is present, and the patient does not use any NSAIDs. Blood pressure in your office was found to be elevated at 160/110 mmg. Recent laboratory testing shows sodium: 146 mmol/L; potassium: 2.4 mmol/L and creatinine: 85 umol/L. There is no proteinuria on usinalycis, Tae most appropriate test to arrange at this time is 2) 24-hour utine collection for aldosterone +) 24-hour urine collection for catecholamines and creatinine ©) 24-hour ambulatory biocd pressure monitor 6) 24-hour urine collection for Cortisal 6) Nuclear medicine Captopril renal scan Question Explanation: “Aldosterone, « hormone produced and secreted by the adrenal glands, signals the kidneys to excrete less sodiam and more potassium, Hyperaldosteronisin can be caused by a tumor (usually a noncancerous adenoma) in the adrenal gland (a Condition called Con's syndrome), although sometimes both glands are Involved and are overactive. Sometimes hyperaldosteronism is a response to certain diseases, such as very high tlood pressure (hypertension) or narrowing of one of the arteries to the kidneys. A doctor who suspects hyperaldosteronism first tests the levels of sodium and potassium in the blood The doctor may also measure aldosterone levels. IFthey are high, spironclactone, a drug thet blocks the action of aldosterone, may be giren to sce £the levels of sodiam and potassium retum to normal. In Conn’s syndrome, the levels of renin are also very low. “Mark this question >= Question Id : 67305 Question 5 of 30 A.48 year old man has persistent hypertension despite taking metoprolol, Enalapril and Nifedipine, Mo history palpitations or fushing is present, and the patient does not use any NSAIDs. Blood pressure in your office was found to be elevated at 160/110 mmg. Recent laboratory testing shows sodium: 146 mmol/L: potassium’ 2.4 mmolL and creatinine: 85 umoV/L. There is no proteimaria on usinalycis, Tae most appropriate test to arrange at this time is Y © 2) 24-hour urine collection for aldosterone +) 24-hour urine collection for catecholamines and creatinine ©) 24-hour ambulatory biocd pressure monitor & 24-hour urine collection for Cortisol 6) Nuclear medicine Captopril renal scan Question Explanation: “Aldosterone, « hormone produced and secreted by the adrenal glands, signals the kidneys to excrete less sodiam and more potassium, Hyperaldosteronisin can be caused by a tumor (usually a noncancerous adenoma) in the adrenal gland (a Condition called Con's syndrome), although sometimes both glands are Involved and are overactive. Sometimes hyperaldosteronism is a response to certain diseases, such as very high tlood pressure (hypertension) or narrowing of one of the arteries to the kidneys. A doctor who suspects hyperaldosteronism first tests the levels of sodium and potassium in the blood The doctor may also measure aldosterone levels. IFthey are high, spironclactone, a drug thet blocks the action of aldosterone, may be giren to sce £the levels of sodiam and potassium retum to normal. In Conn’s syndrome, the levels of renin are also very low. 2i24/2014 2:55:18 PM ‘Mark this question & => Question 6 of 30 All of the following are true regarding diabetic neuropathy EXCEPT 2) Focal or multifocal asymmetric neuropathy is the most common type of neuropathy. ) Dizbetic proximal motor neuropathy is vascular in origin, ©) Manifestations include postural hypotension, incomplete emptying of the bladder and sesmual dysfinction. 4) The most effective approach to diabetic polynenropathy is prevention, ©) Painful diabetic neuropathy can sometimes be alleviated by phenytoin, amitriptyline, and topical capsaicin Question Explanation: ‘The more severe and long-standing is the diabetes, the worse the neuropathy. Distal symmetrical diabetic polyneuropathy is the most common because of is reffactoriness to therapy. Diabetic third nerve palsy. proximal neuropathies. compression neuropathy (carpal tunsel syndrome), end autonomic neuropathies can also occur in long, standing diabetes, Diabetic proximal motor neuropathy is vascular in origin because the blood vessels that supply the motor nerves become damaged and result in progressive nerve ischemia. ‘The Diabetes Control and Complications Trial Research Group showed that intensive treatment with insuin injections delays onset and slows progression of diabetis neuropathy, retinopathy, and nephropathy. Very litle can be dene to restore sencation, but foot ulcers: Osteomyelitis and Charcot joints can be prevented by having a physician check the foot every six months and by keeping the toenals clipped and cleaned. Syinptoms of pain caused by neuropathy can sometimes be alleviated by adjusting the doses of amitriptyine, pheaytoin, desipramine, and topical capsaicin, Topical capsaicin works by intibiting substance P in the superficial nerve Sbers and diminishing the sensation of tenderness. 2i24/2014 2:55:18 PM ‘Mark this question & => Question 6 of 30 All of the following are true regarding diabetic neuropathy EXCEPT Y © a) Focal or multifocal asymmetric neuropathy is the most common type of neuropathy. ) Dizbetic proximal motor neuropathy is vascular in origin, ©) Manifestations incinde postural hypotension, incomplete emptying of the bladder and sesmal dysfunction. 4) The most effective approach ta dichetic polyneuropathy is prevention. 6) Painful diabetic neuropathy can sometimes be alleviated by phenytoin, amitriptyline, and topical capsaicin. Question Explanation: ‘The more severe and long-standing is the diabetes, the worse the neuropathy. Distal symmetrical diabetic polyneuropathy is the most common because of is reffactoriness to therapy. Diabetic third nerve palsy. proximal neuropathies. compression neuropathy (carpal tunsel syndrome), end autonomic neuropathies can also occur in long, standing diabetes, Diabetic proximal motor neuropathy is vascular in origin because the blood vessels that supply the motor nerves become damaged and result in progressive nerve ischemia. ‘The Diabetes Control and Complications Trial Research Group showed that intensive treatment with insuin injections delays onset and slows progression of diabetis neuropathy, retinopathy, and nephropathy. Very litle can be dene to restore sencation, but foot ulcers: Osteomyelitis and Charcot joints can be prevented by having a physician check the foot every six months and by keeping the toenals clipped and cleaned. Syinptoms of pain caused by neuropathy can sometimes be alleviated by adjusting the doses of amitriptyine, pheaytoin, desipramine, and topical capsaicin, Topical capsaicin works by intibiting substance P in the superficial nerve Sbers and diminishing the sensation of tenderness. 224.2014 2:55:41 PM Mark this question & => Question Id : 78560 Question 7 of 30 ‘The bronze skin discoloration associated with hemochromatosis is due to deposition of 2) Melanin. +) Lipofiscin ©) Copper &) Heme 6) Ferritin Question Explanation: ‘Melanin is responsible for the hyperpigmentaiion often seen in hemoctromatosis. None of the other choices is correct, 224.2014 2:55:41 PM Mark this question & => Question Id : 78560 Question 7 of 30 ‘The bronze skin discoloration associated with hemochromatosis is due to deposition of Y © 2 Melann. +) Lipofiscin ©) Copper &) Heme ©) Fersitin Question Explanation: ‘Melanin is responsible for the hyperpigmentaiion often seen in hemoctromatosis. None of the other choices is correct, ‘Marie this question €& => Question Tad : 82228 Question 8 of 30 Which of the following ie NOT associated with inuin resistance? a) Diabetes mellitus 6) Obesity ©) Pineal atrophy syndrome. 4) Acanthosis nigricans. €) Leprechamnism. Question Explanation: Tnsulin resistance may result from preteceptor. receptor, or postreceptot abnotmaities. In diabetics with fall-blown resistance, prereceptor resistance to insulin antibodies is present (A), Obesity the most common cause of insulin resistance, a postreceptor defect, with failure to activate tyrosine kinase, is present (B). Leprechaunism is associated with mutations in the insulin receptor (E) Several smdromes have been associated with acanthosis nigricans. In the type A syndrome affecting young women with reproductive abnormalities, mutations of the inculin receptor have been described. In type B women, insulin resistance results from blocking antibodies to the insulin receptor (D). A rare disorder, pineal hyperplasia syndrome, is associated with insulin resistance (C). ‘Marie this question €& => Question Tad : 82228 Question 8 of 30 Which of the following ie NOT associated with inuin resistance? a) Diabetes mellitus 6) Obesity Y © 0) Pineal atrophy syndrome. 4) Acanthosis nigricans. €) Leprechamnism. Question Explanation: Tnsulin resistance may result from preteceptor. receptor, or postreceptot abnotmaities. In diabetics with fall-blown resistance, prereceptor resistance to insulin antibodies is present (A), Obesity the most common cause of insulin resistance, a postreceptor defect, with failure to activate tyrosine kinase, is present (B). Leprechaunism is associated with mutations in the insulin receptor (E) Several smdromes have been associated with acanthosis nigricans. In the type A syndrome affecting young women with reproductive abnormalities, mutations of the inculin receptor have been described. In type B women, insulin resistance results from blocking antibodies to the insulin receptor (D). A rare disorder, pineal hyperplasia syndrome, is associated with insulin resistance (C). 2/24/2014 2:56:08 PM “Mark this question € => Question Td : 88550, Question 9 of 30 A 28 year old man discovers a mass in his neck. Physical examination reveals a 2 ci mass in one thyroid lobe that does not concentrate radioisotopes on a thyroid scan. Nodule aspiration reveals small "solid balls’ of neoplastic follicular cells. They contan microscopic blood vessels and fibrous stroma in their ceaters. Most likely associated with the diagnosis of this condition is a) Cold intolerance b) Heat intolerance ©) Pretibial msynedenna 4) Radiation therapy ©) Weight gain Question Explanation: The distinctive cell balls described are broken off papillary clusters, and are considered pathognomic of papillary carcinoma of the thyroid. This is the mast common farm oP thyroid carcinoma. It tends to present in the third ta fih decades oflife and shows modest female predominance. Degpite its propensity for local lymphatic intrusion (whish may cause maltfocalty of tumor in the thyroid or cervical lymph node metastases) the tumor generally has an excellent prognosis with 90% of the patients having 20 year survival. Ibis typically associated with a history of ractation to the neck. Pretibial myxedema is associated with Graves's disease, the most common form of kyperthyroidism. Weight gain is associated with hypothyroidism. If anything, the patient's cancer would likely lead to weight lose, Cold intolerance is also asccciated with hypothyroidism. Heat intolerance ic seen with lyperthyxoidiem, ac in poticats with Grave's disease. 2/24/2014 2:56:08 PM “Mark this question € => Question Td : 88550, Question 9 of 30 A 28 year old man discovers a mass in his neck. Physical examination reveals a 2 ci mass in one thyroid lobe that does not concentrate radioisotopes on a thyroid scan. Nodule aspiration reveals small "solid balls’ of neoplastic follicular cells. They contan microscopic blood vessels and fibrous stroma in their ceaters. Most likely associated with the diagnosis of this condition is a) Cold intolerance 'b) Heat intolerance c) Pretibial myxedema Y © 4) Radiation therapy 2) Weight gain Question Explanation: The distinctive cell balls described are broken off papillary clusters, and are considered pathognomic of papillary carcinoma of the thyroid. This is the mast common farm oP thyroid carcinoma. It tends to present in the third ta fih decades oflife and shows modest female predominance. Degpite its propensity for local lymphatic intrusion (whish may cause maltfocalty of tumor in the thyroid or cervical lymph node metastases) the tumor generally has an excellent prognosis with 90% of the patients having 20 year survival. Ibis typically associated with a history of ractation to the neck. Pretibial myxedema is associated with Graves's disease, the most common form of kyperthyroidism. Weight gain is associated with hypothyroidism. If anything, the patient's cancer would likely lead to weight lose, Cold intolerance is also asccciated with hypothyroidism. Heat intolerance ic seen with lyperthyxoidiem, ac in poticats with Grave's disease. 2i24/2014 2:56:23 PM ‘Mark this question & => Question Td : 89851 Question 10 of 30 A 34 year old men presents with decreased libido, According to him he did not start shaving until he was 18 and now needs to shave only every other day. He has virtually no secondary sexual hair. His testes are small and atrophic and skin is soft and pale. Which of the following is the most lkely diegnosis? 8) Elinefelter‘s syndrome. b) Nonsecreting chromophobe tumor of the pitatary, ©) Secretory pituitary tumor. 6) Pseudotumor cerebsi ©) Germinoma Question Explanation: Non secreting chromophobe tumors of the pituitary are often associated with sof, smooth skin, ebsence of secondary sexual hair, and small and atrophic testes. Klinefelter‘s syndrome (KY) produces small testes, inferilty and, often, gynecomastia Secondary sexaial hair is reduced. The skin may become quite wrinkled A secretory pinitary tumor would not produce a clinical syndrome such as this, nor would pseudotumor cerebri (beniga increased intracranial pressure) or Germinoma 2i24/2014 2:56:23 PM ‘Mark this question & => Question Td : 89851 Question 10 of 30 A 34 year old men presents with decreased libido, According to him he did not start shaving until he was 18 and now needs to shave only every other day. He has virtually no secondary sexual bair. His testes are small and atrophic and skin is coft and pale. Which of the following is the most lkely diagnosis? 8) Elnefeltee's eyndrome. Y © b) Noasecreting chromophobe tumor of the pituitary. 6) Secretory pituitary tumor. 6) Pseudotumor ceretri ©) Germinoma Question Explanation: Non secreting chromophobe tumors of the pituitary are often associated with sof, smooth skin, ebsence of secondary sexual hair, and small and atrophic testes. Klinefelter‘s syndrome (KY) produces small testes, inferilty and, often, gynecomastia Secondary sexaial hair is reduced. The skin may become quite wrinkled A secretory pinitary tumor would not produce a clinical syndrome such as this, nor would pseudotumor cerebri (beniga increased intracranial pressure) or Germinoma 2i24/2014 2:56:35 PM ‘Marke this question & => Question Id : 93211 Question 11 of 30 Obesity is commonly associcted with which one of the following? a) Facial edema ') Insulin resistance. c) Hyperthyroidism. 4) Addison’s dsease ©) Hypernatremia. newer (UERARAN omer sors Explanation Repos An Error Question Explanation: Obesity produces a relative insulin resistance and hyperinsulinemia, at least early in the development of diabetes. Facial edema (A) is not associated with obesity nor are hyperthyroidism (C), Addison's disease (D), or hypematremia E). 2i24/2014 2:56:35 PM ‘Marke this question & => Question Id : 93211 Question 11 of 30 Obesity is commonly ascocisted with which one of the following? a) Facial edema ¥ © b) Insulin resistance. c) Hyperthyroidism. 4) Addison's disease e) Hypernatremia. newer (UERARAN omer sors Explanation Repos An Error Question Explanation: Obesity produces a relative insulin resistance and hyperinsulinemia, at least early in the development of diabetes. Facial edema (A) is not associated with obesity nor are hyperthyroidism (C), Addison's disease (D), or hypematremia E). ‘Mark this question & => Question Td : 93286 Question 12 of 30 A.30 year old woman has sudden convulsions and paresthesias of the lips and extremities. She also has extensive spasm of her (arms, causing cramps. There is contraction of the facial muscle in response to tapping the facial nerve against the bone anterior to the ear. ECG shows a prolonged QT interval, Which of the folowing is NOT associated with this condition? a) Chronic renal insufficiency b) Hypoparathyroidism. ¢) Malabsorption. 4) Sarcoidosis. ©) Sepsis Anewor (UBRIRNRIEN) omer veers Explanation Report An ror Question Explanation: This patient has symptoms ofhypocalsemia. This patient has Chvostek‘s sign with spasm of the facial muscle. Prolongation of the QT snterval can cause torsades de pointes, a ventricular arrythmia Sarcoidosis causes hypercalcemia via an increase in hydroxylation of vitamin D by the granulomas and, thus, an increase in calcium absorption. Chronic renal insufficiency (A) causes hypocalcemia because of inadequate 1-hydroxylation of vitamin D by the kidneys and thns a decrease in absorption of calcium via the GT tract Hypoparathyroidism (B) is secondary to decrease in parathyroid hormone and, thus, decrease in bone resorption and hypocalcemia, Malabsorption (C) causes hypocalcemia via decreased absorption via the Gl tract, Examples are pancreatic insufficiency and gluten- sensitive enteropathy. Sepsis (E) causes 20% of hypocalcemic symptoms in Gram-negative sepsis. These patients have @ high mortality rate, This is secondary to defects in the parathyroid vitamin D axis. ‘Mark this question & => Question Td : 93286 Question 12 of 30 ‘A 30 year old woman has sudden convnisions and paresthesias of the lips and extremities. She also has extensive spasm ofher (arms, causing cramps. There is contraction of the facial muscle in response to tapping the facial nerve against the bone anterior to the ear. ECG shows a prolonged QT interval. Which of the following is NOT associated with this condition? a) Chronic renal insufficiency. b) Hypoparathyroidism. ) Malabsorption. Y © A) Sarcoidosis. 2) Sepsis, Anewor (UBRIRNRIEN) omer veers Explanation Report An ror Question Explanation: This patient has symptoms ofhypocalsemia. This patient has Chvostek‘s sign with spasm of the facial muscle. Prolongation of the QT snterval can cause torsades de pointes, a ventricular arrythmia Sarcoidosis causes hypercalcemia via an increase in hydroxylation of vitamin D by the granulomas and, thus, an increase in calcium absorption. Chronic renal insufficiency (A) causes hypocalcemia because of inadequate 1-hydroxylation of vitamin D by the kidneys and thns a decrease in absorption of calcium via the GT tract Hypoparathyroidism (B) is secondary to decrease in parathyroid hormone and, thus, decrease in bone resorption and hypocalcemia, Malabsorption (C) causes hypocalcemia via decreased absorption via the Gl tract, Examples are pancreatic insufficiency and gluten- sensitive enteropathy. Sepsis (E) causes 20% of hypocalcemic symptoms in Gram-negative sepsis. These patients have @ high mortality rate, This is secondary to defects in the parathyroid vitamin D axis. ‘Mark this question & => jan Id : 96790 Question 13 of 30 “Whick of the following condition is NOT associated with decreased testosterone effects in males? a) Adenocarcinoma of the lung ) Hemeckromatosis ©) Chronic hemodialysis 4) Sickle cell anemia. €) Citthosis of the liver Question Explanation: Adenocarcinoma of the ling is not specifically associated with diminished testosterone effects. Hemochromatosis, chronic hemodialysis, sickle cell anemia, and cirrhosis of the liver can all be associated with underviilization in men, ‘Mark this question & => jan Id : 96790 Question 13 of 30 “Whick of the following condition is NOT associated with decreased testosterone effects in males? Y © a) Adenocarcinoma of the lung, ) Hemeckromatosis ©) Chronic hemodialysis 4) Sickle cell anemia. €) Citthosis of the liver Question Explanation: Adenocarcinoma of the ling is not specifically associated with diminished testosterone effects. Hemochromatosis, chronic hemodialysis, sickle cell anemia, and cirrhosis of the liver can all be associated with underviilization in men, 22472014 2:57:13 PM ‘Mark this question & => Question Id : 98048 Question 14 of 30 ‘The hormone that does NOT play arcle in protecting against hypoglycemia is a) Glucagon ) Epinephrine c) Somatomedin ) Norepinephrine e) Cortisol Question Explanation: Counterregulatory hormones increase hepatic glucose production and decrease utilization in nonhepatic tissues. Glucagon, secreted by pancreatic alpha cells, is the psimary counter regulatory hormone, Epinephnne ard norepinephrine, released fom the adrenal medulla and sympathetic nervous system, are critical in the absence of glucagon, Cortisol and growth hormone play an important role during periods of prolonged fasting or sustained hypoglycemia, Somatomedans, also known as insulin lice growth factors, are jiuportant in growth stimulation. 22472014 2:57:13 PM ‘Mark this question & => Question Id : 98048 Question 14 of 30 ‘The hormone that does NOT play arcle in protecting against hypoglycemia is a) Glucagon ) Epinephrine Y © ©) Somatomedin ) Norepinephrine ©) Cortisol Question Explanation: Counterregulatory hormones increase hepatic glucose production and decrease utilization in nonhepatic tissues. Glucagon, secreted by pancreatic alpha cells, is the psimary counter regulatory hormone, Epinephnne ard norepinephrine, released fom the adrenal medulla and sympathetic nervous system, are critical in the absence of glucagon, Cortisol and growth hormone play an important role during periods of prolonged fasting or sustained hypoglycemia, Somatomedans, also known as insulin lice growth factors, are jiuportant in growth stimulation. 2i24/2014 2:57:26 PM ‘Marke this question <& => Question Td = 109921 Question 15 of 30 All ofthe following are characteristics oftthe low'T;, normal Ty vaviart of sick euthyroid eynérome, EXCEPT a) Decreased production of Ty resilts from extratkyroidal T, conversion +b) Decreased total T; cotrelates with the seventy of systemic ines. ©) Nommal TSH levels help to exclude hypottyroicisrn 4) Serum #1 concentrations are decreased ©) Serum rT metabolism to 1Ty is impaired. Anewor (UEQPIRISVEN) ones veers Explanation Report An Error Question Explanation: ‘A decrease in T; levels is accompanied by an increase in serum rT; used by a decrease in plasma clearance of rT; due to inhibition ofits 5’-monodeiodination. The low Ts, normal T, variant, is the most fequently encountered variant of the sick euthyroid syndrome, ‘The decrease in serum Ty correlates roughly with the severity of the underlying systemic ilhess. The decreased production of Ty results from impairment of extrathyroidal conversion of Ty to Tz, Nomnal serum T, and TSH levels distinguish this syndrome from hypothyroidism. 2i24/2014 2:57:26 PM ‘Marke this question <& => Question Td = 109921 Question 15 of 30 All ofthe following are characteristics oftthe low'T;, normal Ty vaviart of sick euthyroid eynérome, EXCEPT a) Decreased production of T results from extrathyroidal Ty conversion. +b) Decreased total T; cotrelates with the seventy of systemic ines. ©) Nommal TSH levels help to exclude hypottyroicisrn V © 4) Serum 1T3 concentrations are decreased. ©) Serum rT metabolism to 1Ty is impaired. Anewor (UEQPIRISVEN) ones veers Explanation Report An Error Question Explanation: ‘A decrease in T; levels is accompanied by an increase in serum rT; used by a decrease in plasma clearance of rT; due to inhibition ofits 5’-monodeiodination. The low Ts, normal T, variant, is the most fequently encountered variant of the sick euthyroid syndrome, ‘The decrease in serum Ty correlates roughly with the severity of the underlying systemic ilhess. The decreased production of Ty results from impairment of extrathyroidal conversion of Ty to Tz, Nomnal serum T, and TSH levels distinguish this syndrome from hypothyroidism. ‘Mark this question & => Question Td : 119797 Question 16 of 30 “Which of the folowing drug CANNOT affect the gincose tolerance test? a) Nicotine, ») Ethanol ) Caffeine. 4) Acetaminophen 2) Aspirin, Anowor [UERRNIEN otnorueors Explanation Report An Error Question Explanation: Acetaminophen does not affect the glucose tolerance test, Nicotine and caffdine can produce apparent hyperglycemia during a glucose tolerance test, while ethanol and aspirin can produce low glucose levels, ‘Mark this question & => Question Td : 119797 Question 16 of 30 “Which of the folowing drug CANNOT affect the gincose tolerance test? a) Nicotine, ») Ethanol ) Caffeine. Y © d) Acetaminophen 2) Aspirin, Anowor [UERRNIEN otnorueors Explanation Report An Error Question Explanation: Acetaminophen does not affect the glucose tolerance test, Nicotine and caffdine can produce apparent hyperglycemia during a glucose tolerance test, while ethanol and aspirin can produce low glucose levels, Mark this question & => Question Td : 120159 Question 17 of 30 A.37 year old man with along history of alcohel intake comes to the physician's office complaining of painful breasts which are enlarged. Which of the following is NOT a cause of gynecomastia? 2) Cirthosis of the liver. +) Spironolactone. ©) Thyrotoxicosis, 4) Carcinoma of the prostate. ©) Germ cell testicular tumors, Question Explanation: Carcinoma of the prostate gland is the third most common cause of morbidity from cancer in men, It is usually adenocarcinomas and does not cause gmecomastia unless hormone therapy is given, Citthosis of the liver causes hyperestrogenemia due to decreased hepatic extraction of androstenedione, leading to increased conversion of estrone to estradiol, leading to gmecomastia Spironolactone, cimetidine, digitalis, and phenothiazines are just some of the drugs that cause gynecomastia Thyrotaxicosis canses increased production of androstenedione, which is converied to estradiol, and leads to gynccomastia. Germ call testicular tumors cause gynecomastia by secreting human chorionic gonadotropin, which in turn increases testicular secretion of estradiol Mark this question & => Question Td : 120159 Question 17 of 30 A. 37 year old man with aloag history of alcehel intake comes to the physician's office complaining of painful breasts which are enlarged. Which of the following is NOT a cause of gynecomastia? 2) Cirthosis of the liver. +) Spironolactone. ©) Thyrotoxicesis Y © 6) Carcinoma of the prostate ©) Germ cell testicular tumors, Question Explanation: Carcinoma of the prostate gland is the third most common cause of morbidity from cancer in men, It is usually adenocarcinomas and does not cause gmecomastia unless hormone therapy is given, Citthosis of the liver causes hyperestrogenemia due to decreased hepatic extraction of androstenedione, leading to increased conversion of estrone to estradiol, leading to gmecomastia Spironolactone, cimetidine, digitalis, and phenothiazines are just some of the drugs that cause gynecomastia Thyrotaxicosis canses increased production of androstenedione, which is converied to estradiol, and leads to gynccomastia. Germ call testicular tumors cause gynecomastia by secreting human chorionic gonadotropin, which in turn increases testicular secretion of estradiol 2i24/2014 2:58:03 PM ‘Mark this question & => Question 18 of 30 A.23 year old female has never menstruated, Her pubic and axillary hairs are absent, Her breasts are stnall and a uterus is not palpable. Karyotype is 46, -Y. DNA analysis shows point mutation in one of the codons in the androgen receptor gene. Gonads are removed after exploratory surgery. Histologic exam of the gonads reveal the preseace of seminiferous tubules with normal Leydig cells. Prior to surgery (when compared to that of a normal woman) labs would have shown which of the following? a) High testosterone; high LE b) High testosterone; low LH c) Low testosterone; high LET 4) Low testosterone; low LH ©) Normal testosterone; normal LH Avewor [UEIRNRNEN) Otor Users Explanation Report An Error Question Explanation: DMA analysis shows that this patient is a genotypic male with an abnormality ofthe androgen receptor molecule, The androgen receptor molecule normaly combines with testosterone inside the cell, and the complex binds with chromatin, resulting in she synthesis ofmessenger RNA. Because the target tissues contain detective androgen receptor protein, they are unable to respond to testosterone Ancrogen insenstivity during the fetal period means that male sexual developmentis impaired and female extemal genitalia are formed, Because the androgen receptor in hypothalamic and pituitary tissue is also defective, normal testosterone suppression of pituitary gonadotrophs, such as Iuteiniang hormone (LH), is absent. As a result, serum levels of gonadotropins are elevated resulting in increased production of testosterone by Leydig cells. High testosterone and low LH would not be expected in the androgen insensitivity syndrome. Cells of the pituitary and hypothalamus also contain the androgen receptor protein. Feedback suppression of LH secretion occurs wien the molecular complex is present in the hypothalamus and pituitary. Testosterone is low and LH is high in primary hypogonadism due to detective gonadal steroid synthesis. The reduced testosterone results in feedback elevation of LE secretion by the pituitary Low TH insulting in depressed levels of testosterone secretion is characteristic of a detect in hypothalamic pitutary function (hypogonadotrophic hypogonadism), 2i24/2014 2:58:03 PM ‘Mark this question & => Question 18 of 30 A.23 year old female has never menstruated. Her pubic and axillary hairs are absent, Her breasts are small and a uterus is not palpable, Karyotype is 46, XY. DNA analysis shows point mutation in one of the codons in the androgen receptor gene, Gonads are removed after exploratory surgery. Histologic exam of the gonads reveal the presence of ssminiferous tubules with normal Leydig cells. Prior to surgery (when compared to that of a normal woman) labs would have shown which of the following? JY © a) High testosterone; high LH +b) High testosterone; low LH o) Low tectosterone; high LE 4) Low testosterone; low LH ©) Normal testosterone; normal LH Avewor [UEIRNRNEN) Otor Users Explanation Report An Error Question Explanation: DMA analysis shows that this patient is a genotypic male with an abnormality ofthe androgen receptor molecule, The androgen receptor molecule normaly combines with testosterone inside the cell, and the complex binds with chromatin, resulting in she synthesis ofmessenger RNA. Because the target tissues contain detective androgen receptor protein, they are unable to respond to testosterone Ancrogen insenstivity during the fetal period means that male sexual developmentis impaired and female extemal genitalia are formed, Because the androgen receptor in hypothalamic and pituitary tissue is also defective, normal testosterone suppression of pituitary gonadotrophs, such as Iuteiniang hormone (LH), is absent. As a result, serum levels of gonadotropins are elevated resulting in increased production of testosterone by Leydig cells. High testosterone and low LH would not be expected in the androgen insensitivity syndrome. Cells of the pituitary and hypothalamus also contain the androgen receptor protein. Feedback suppression of LH secretion occurs wien the molecular complex is present in the hypothalamus and pituitary. Testosterone is low and LH is high in primary hypogonadism due to detective gonadal steroid synthesis. The reduced testosterone results in feedback elevation of LE secretion by the pituitary Low TH insulting in depressed levels of testosterone secretion is characteristic of a detect in hypothalamic pitutary function (hypogonadotrophic hypogonadism), 2242014 2:58:16 PM ‘Mark this question & => Question Id : 179847 Question 19 of 30 A patient has severe arthritis involving the lower back. Before making a diagnosis of ankylosing spondylitis, he should be questioned about which of the following diseases? a) Careinoid syndrome b) Celiac disease c) Crohn disease & Peptic ulcer ©) Whpple disease Question Explanation: ‘Ten to twenty percent of patients with Crohn disease and ulcerative colis develop an arthntis that resembles ankylosing spondylitis. Similar arthropathies are seen in psosiasis or Reiter syndrome (arthritis. urethritis, conjunctivitis. and rash following chlarnydial infection), as well as related syndromes seen following Shigella, Salmonella, or Yersinia enterocolitis. The other answers are distracters. 2242014 2:58:16 PM ‘Mark this question & => Question Id : 179847 Question 19 of 30 A patient has severe arthritis involving the lower back. Before making a diagnosis of ankylosing spondylitis, he should be questioned about which of the following diseases? a) Carcinoid syndrome ) Celiac disease ¥ © ¢) Crohn disease ©) Peptic uleer ©) Whpple disease Question Explanation: ‘Ten to twenty percent of patients with Crohn disease and ulcerative colis develop an arthntis that resembles ankylosing spondylitis. Similar arthropathies are seen in psosiasis or Reiter syndrome (arthritis. urethritis, conjunctivitis. and rash following chlarnydial infection), as well as related syndromes seen following Shigella, Salmonella, or Yersinia enterocolitis. The other answers are distracters. 2124/2014 2:58:26 PM “Mark tis question => Question 20 of 30 A.61 year female has vague aches and pains. She has a family Listory of osteoporosis. She is 10 years post menopausal but hes not taken any female HRT. Dual energy X-ray absorptiometry density measured by DEXA would signify osteopaenia at a measured site? a) AT ccore of 26 d)AT score of -1.8 8) Az evore of 2.0 a) Az score of -1.5 2) AT score of -09 Question Explanation: Osteopeenia is defined as a T score off between -1 and -2.5 standard deviations below the bone mineral density ofa young fernale Osteoporosis is defined as <-2.5 SD. These measurements are imp ortent as they signify a greatly increased risk of facture. Z scores, to the bone mineral density compared with that of a ‘normal’ age matched subject. 2124/2014 2:58:26 PM “Mark tis question => Question 20 of 30 A 61 year female has vague aches and pains. She has a family history of osteoporosis. She is 10 years post menopausal but has not taken any female HRT. Dual energy X-ray absorptiometry density measured by DEXA would signify osteopaenia at a measured site? a) AT score of 26 Y © d)AT score of -18 2) Az score of 2.0 Az score of -15 2) AT score of -0.9 Question Explanation: Osteopeenia is defined as a T score off between -1 and -2.5 standard deviations below the bone mineral density ofa young fernale Osteoporosis is defined as <-2.5 SD. These measurements are imp ortent as they signify a greatly increased risk of facture. Z scores, to the bone mineral density compared with that of a ‘normal’ age matched subject. ‘Marke this question & => Question Td : 195792 Question 21 of 30 A48 year old woman is newly diagnosed with type 2 dicbetes. She has a history of hypertension and sleep apnoea On exam her BP is 149/2mmHg, and her BML is 39. Her creatinine is 139 pmol (60-120) and fasting glucose is 9.0 mmoll (<7.0). According to the ADA/EASD consensus, the most appropriate therepy for her is, a) Acarbose +b) Intensive diet and lifestsle advice ) Tntensive diet and lifestyle advise with metformin 4) Pioglitazone @) Referral for bariatric Question Explanatioy Tntensive diet and lifestyle advice with metformin started concurrently is the consensus recommendation, Some confision here may come from the 8th option above, referral for bariatric surgery. Whilst the authors acknowledge that bariatvic surgery may be associated with weight loss of 20 kg or more, and reversal of metabolic abnormaites, they stop short of making a recommendation qualified by body mess index (EMI in the guidelines. Whilst acarbose may sesultin a small degree of weight loss and modest reduction in HbA tc, metformin is recommended ahead oft. Pioglitazone canses weight gain and increases the risk af bone fracture- as such it would not be recommended as intial therapy here ‘Marke this question & => Question Td : 195792 Question 21 of 30 48 vear old woman is newly diagnosed with type 2 dizbetes. She has a history of hypertension and sleep apnoea, On exam her BP is 149/82mmaHg, and her BMI is 39. Her crectinine is 139 pmol (60-120) and fasting glucose ie 9.0 miaoll (<7.0). According to the ADAEASD consensus, the most appropriate therapy for her is a) Acarbose 'b) Intensive diet and lifestyle advice SV © ©) Intensive diet and lifestyle advise with metformin, 4) Pioglitazone ) Referral for bariatric Question Explanatioy Tntensive diet and lifestyle advice with metformin started concurrently is the consensus recommendation, Some confision here may come from the 8th option above, referral for bariatric surgery. Whilst the authors acknowledge that bariatvic surgery may be associated with weight loss of 20 kg or more, and reversal of metabolic abnormaites, they stop short of making a recommendation qualified by body mess index (EMI in the guidelines. Whilst acarbose may sesultin a small degree of weight loss and modest reduction in HbA tc, metformin is recommended ahead oft. Pioglitazone canses weight gain and increases the risk af bone fracture- as such it would not be recommended as intial therapy here 2i24/2014 22 ‘Mark this question 3:52 PM ¢=> Question 22 of 30 Question Td : 195924 24 year old woman has weight gain, oligomenorhoes and primary infertility. She takes Lithium for bipolar disorder. On exam her BMLis 32kg/m2. Labs reveal fice T4 of 6 4 pmol (10-22), TSH of 42 mU/L (0.4-5) and prolactin of 980 mUIL (50-450). The most appropriate treatment for this patient is 3) Cobergoline ) Cabergoline plus thyroxine c) Metformin 4) Thyroxine ) Stop Lithium Question Explanation: ‘This patient has primary hypothyroidism which would explain the increasing weight and the associated hyperprolactinaemia. The latter occurs as a consecunece of reduced dopaminergic tone. The most appropriate treatment for her would be thyroxine which would through euthyroiciom be expected to normalise prolactin concentration. In tum this may improve weight, menstrual function and ‘erty. 2i24/2014 22 ‘Mark this question 3:52 PM ¢=> Question 22 of 30 Question Td : 195924 24 year old woman has weight gain, oligomenorhoes and primary infertility. She takes Lithium for bipolar disorder. On exam her BMLis 32kg/m2. Labs reveal fice T4 of 6 4 pmol (10-22), TSH of 42 mU/L (0.4-5) and prolactin of 980 mUIL (50-450). The most appropriate treatment for this patient is 3) Cobergoline ) Cabergoline plus thyroxine c) Metformin Y © dD) Thyroxine ) Stop Lithium Question Explanation: ‘This patient has primary hypothyroidism which would explain the increasing weight and the associated hyperprolactinaemia. The latter occurs as a consecunece of reduced dopaminergic tone. The most appropriate treatment for her would be thyroxine which would through euthyroiciom be expected to normalise prolactin concentration. In tum this may improve weight, menstrual function and ‘erty. Mark this question e& => Question Td : 195964 Question 23 of 30 A.55 year old man has a5 year history of increased sweats and change in shoe size. Exam reveals prognathisi and mactoglossia, with large hands. Blood pressure is 180/94 miaFig but visual fielé examination is fullto confrontation. The test thet would be agnostic ie which one of the following? 2) Oral glucose tolerance test 6) TRH test ©) Insulin tolerance test 4) Pituitary MRI ©) IGF-1 concertration Anower [UEQINSRN) osrer User's Explanation Report An Eror Question Explanation: ‘The diagnosis of acromegaly is confirmed with a failure of GH Suppression during an oral ghicose tolerance test: Though a Pituitary adenoma may be present itis not diagnostic of acromegaly. Mark this question e& => Question Td : 195964 Question 23 of 30 A.55 year old man has a5 year history of increased sweats and change in shoe size. Exam reveals prognathisi and mactoglossia, with large hands. Blood pressure is 180/94 miaFig but visual fielé examination is fullto confrontation. The test thet would be gnostic ie which one of the following? Y © a Oral ducose tolerance test 6) TRH test ©) Insulin tolerance test 4) Pituitary MRI ©) IGF-1 concertration Anower [UEQINSRN) osrer User's Explanation Report An Eror Question Explanation: ‘The diagnosis of acromegaly is confirmed with a failure of GH Suppression during an oral ghicose tolerance test: Though a Pituitary adenoma may be present itis not diagnostic of acromegaly. 2124/2014 2:59:21 PM “Mark this question => Question 24 of 30 A.47 year olé woman with a recent dagnosis of type 2diaberesis seen because of deranged liver function rests (LETS). She dtinks, at most, 4 units of alcohol weekey. On exam she is obese with a BMI of 38.9 keg/in2 and her recent LFTs stow: facr 140 UA (S-40) [pst 150 Wal|(10-40) [Alkaline phosphatase)[250 Ui [43-103 ‘The most likely cause of this derangement is which one of the following? a) Alcoholic iver disease ») Autoimmune hepatitis ¢) Drug induced hepatitis ® Hleemochromatosis £) Non-alcoholis steatohepatitis Question Explanation: Diebctes mellitus associated with cbesity is the most likely cause of non alcoholic fatty liver disease (NAFLD) in this patient, which is caused by fatty accumulation in the liver leading to inflammation Other causes of hepatitis need to be exchided before making this diagnosis. Patients who are obese and diabetic are advised to lose weight and control their diabetes. Usually, a low fat, low calorie diet is recommended along treatment to lower HbA 1c. Patients with NAFLD should avoid alcohol or other substances that could be harmfil to the liver. The differential diagnosis could include haemochromatosis although this is unlicely in a menstraant female dae to blood losses and there is nothing else in the history to suggest this diagnosis, 2124/2014 2:59:21 PM “Mark this question => Question 24 of 30 A.47 year olé woman with a recent dagnosis of type 2diaberesis seen because of deranged liver function rests (LETS). She dtinks, at most, 4 units of alcohol weekey. On exam she is obese with a BMI of 38.9 keg/in2 and her recent LFTs stow: facr 140 UA (S-40) [pst 150 Wal|(10-40) [Alkaline phosphatase)[250 Ui [43-103 ‘The most lcely cause of this derangement is which one of the following? a) Alcoholic liver disease ») Autoimmune hepatitis ) Drug induced hepatitis ® Hleemochromatosis Y © 2) Non-alcoholic steatohepatitis Question Explanation: Diebctes mellitus associated with cbesity is the most likely cause of non alcoholic fatty liver disease (NAFLD) in this patient, which is caused by fatty accumulation in the liver leading to inflammation Other causes of hepatitis need to be exchided before making this diagnosis. Patients who are obese and diabetic are advised to lose weight and control their diabetes. Usually, a low fat, low calorie diet is recommended along treatment to lower HbA 1c. Patients with NAFLD should avoid alcohol or other substances that could be harmfil to the liver. The differential diagnosis could include haemochromatosis although this is unlicely in a menstraant female dae to blood losses and there is nothing else in the history to suggest this diagnosis, 2242014 2:59:33 PI ark this question ec: Question 25 of 30 A 25 year old woman presents with episodes of dizziness mainly on standing, Her investigation shows hyperkalaemic acidosis. She is most likely suffering ffom which undetlying condtion? a) Cushing's eyndrome b) Addison's disease ©) Conn's syndrome 4) Type | renal tubular acidosis ©) Bulimia nervosa Question Explanation: Her symptoms are suggestive of postural hypotension, which together with hyperkalaemic (and hyponatraemia) acidosis would strongly indicate the presence of Addison’s disease. Cushing's and Conn’s syndromes are associated with hypertension and hypokalaemia, Hypokalaemia is the most frequeat complication of bulimia which may cause cardiac arhythmias, fits and paraesthesia, Renal tubular acidosis (RTA) is due to inability of the renal tubules to maintain acid-base balance, causing a hyperchloraemia and a normel anion-gap. In type 1 (distal) RTA; there is hypokalaemic acidosis with low urinary ammonium production Patients present with hyperventilation/acidosis and muscular weakness from hypokalaemia Ta type 4 RTA (hyporeninaemic hypoaldosteronism), there is hyperlcalaemic acidosis caused by chronic renal insufficiency from diabetes or tubuloincerstitial disease. 2724/2014 2:59:33 PM ‘Mark this question & => Question 25 of 30 ‘A.25 year old woman presents with episodes of dizziness mainly on standing. Her investigation shows hyperkalaemic acidosis. She is most lcely suffering ffom which underlying condition? a) Cushing's syndrome Y © b) Addison's disease c) Conn‘s syndrome d) Type | renal tubular acidosis e) Bulimia nervosa Question Explanation: Her symptoms are suggestive of postural hypotension, which together with hyperkalaemic (and hyponatraemia) acidosis would strongly indicate the presence of Addison’s disease. Cushing's and Conn’s syndromes are associated with hypertension and hypokalaemia, Hypokalaemia is the most frequeat complication of bulimia which may cause cardiac arhythmias, fits and paraesthesia, Renal tubular acidosis (RTA) is due to inability of the renal tubules to maintain acid-base balance, causing a hyperchloraemia and a normel anion-gap. In type 1 (distal) RTA; there is hypokalaemic acidosis with low urinary ammonium production Patients present with hyperventilation/acidosis and muscular weakness from hypokalaemia Ta type 4 RTA (hyporeninaemic hypoaldosteronism), there is hyperlcalaemic acidosis caused by chronic renal insufficiency from diabetes or tubuloincerstitial disease. ‘Mark this question = => Question Td : 196426 Question 26 of 30 A.25 year old male body builder and his wife have been trying to conceive for 3 years. He is found to be azospermic. MRI of the pituitary shows no abnormality. LHis <1.0 TIWL (3 6 -17.1), FSH is <1.0 TUM (2.25 -20) and testosterone is 16.0 nmolfl (9 -34.7). ‘The likely clagnosis is a) Anabolic steroid use bv) Androgen insensitivity syndrome ) Kallman’s syndrome 4) Non functioning pituitary adenoma ) Testicular teratoma Question Explanation: ‘The most likely diagnosis is steroid induced hypogonadism. Body builders may be involved in the ilicit use of anabolic and androgenic steroids. These results are consistent with ongoing use of androgens. The hypogonadism if persistent may be treated with funan chorionic gonadotropin. Ih the event of a non-functioning pituitary tumour, the testosterone would be low together with the LET and FSE, and an MRI of the pititary would not miss this diagnosis. The GH axis would also be likely to be suppressed, and a low IGF-1 would result. In the event of androgen insensitivity, the patient may aopear phenotypically female. One would expect a low testosterane in isolated gonadotraphin deficiency. Kalknan’s syndrcme results in hypagonadotraphic hypagonadism. A teratorna is unlikely to cause hypegonaclotroghic hypogonadism. ‘Mark this question = => Question Td : 196426 Question 26 of 30 A.25 year old male body builder and his wife have been trying to conceive for 3 years. He is found to be azospermic. MRI of the pituitary shows no abnormality. LHis <1.0 TIWL (3 6 -17.1), FSH is <1.0 TUM (2.25 -20) and testosterone is 16.0 nmolfl (9 -34.7). ‘The likely diagnosis is Y © a) Anabolic steroid use bv) Androgen insensitivity syndrome ) Kallman’s syndrome 4) Non functioning pituitary adenoma ) Testicular teratoma Question Explanation: ‘The most likely diagnosis is steroid induced hypogonadism. Body builders may be involved in the ilicit use of anabolic and androgenic steroids. These results are consistent with ongoing use of androgens. The hypogonadism if persistent may be treated with funan chorionic gonadotropin. Ih the event of a non-functioning pituitary tumour, the testosterone would be low together with the LET and FSE, and an MRI of the pititary would not miss this diagnosis. The GH axis would also be likely to be suppressed, and a low IGF-1 would result. In the event of androgen insensitivity, the patient may aopear phenotypically female. One would expect a low testosterane in isolated gonadotraphin deficiency. Kalknan’s syndrcme results in hypagonadotraphic hypagonadism. A teratorna is unlikely to cause hypegonaclotroghic hypogonadism. 2/24/2014 2:59:56 PM “Mark this question => Question Id : 196498 Question 27 of 30 ‘Which one of the following statement is correct regarding diabetic retinopathy? a) Microaneurysms represent secular dilatction of retinal arterioles ) Hard exudates represent calcium depostes in the retina ¢) Cotton wool spot represert infarcts of the nerve fibre layer of the retina ) Haemotrhages close to the fovea are not potentially sight threatening 2) Laser photocoagulation is applied directly to new vessels to destroy them Question Explanation: MAS are capillery aneurysms. Haemorrhages are collections of exudated lipid and protein, Cis correct; multiple CWS are a pre- proliferative sign Haemorrhages (cr HE) close to the fovea represent a rise of macular oedema and are therefore sight threatening Laser destroys ischaemic tut vieble retina to reduce the secretion of angiogenic growth factors and allow new wesel regresion, itis aot applied directly to new vessels as this would cause bleeding 2/24/2014 2:59:56 PM “Mark this question => Question Id : 196498 Question 27 of 30 ‘Which one of the following statement is comect regarding ctabetic retinopatiy? 8) Microaneurysms represent secular dilatation of retinal arterioles 6) Hard exudates represent calcium deposites in the retina ¥ © ©) Cotton wool spot represent infarcts of the nerve fibre layer of the retina 4) Heemorthages close to the fovea are not potentially sight threatening 2) Laser photocoagulation is applied directly to new vessels to destroy them Question Explanation: MAS are capillery aneurysms. Haemorrhages are collections of exudated lipid and protein, Cis correct; multiple CWS are a pre- proliferative sign Haemorrhages (cr HE) close to the fovea represent a rise of macular oedema and are therefore sight threatening Laser destroys ischaemic tut vieble retina to reduce the secretion of angiogenic growth factors and allow new wesel regresion, itis not applied directly to new vessels as this vould cause bleeding 2i24/2014 3:00.07 PM “Mark this question & => Question Td : 196710 Question 28 of 30 4.22 year old woman with type 1 diabetes (TIDM) of three years duration presents with weight loss. She has normal menstrual cycles and bowel habits are unchanged. Her EMT is 23 kg/in2 and lab studies reveal « haemoglobin of 7 g/dL (11.5-16.5) and a MCV of 69 £L (80-96). The most lkely diggnosis is a) Anorexia nervosa ) Bacterial overgrowth ©) Beta-thelassaetia minor 4) Coeliac disease €) Crohn's disease Question Explanation: Coeliac disease is the likely option as this patient has autoimmnne disease (IDM), an iron deficiency anaemia and litle in the why of symptoms, Bacterial overgrowth is associated with profuse diarrhoea and a macrocytosis due to vitarrin B12 consuraption. Crohn's disease agein would be expected to be symptomatic. 2i24/2014 3:00.07 PM “Mark this question & => Question Td : 196710 Question 28 of 30 4.22 year old woman with type 1 diabetes (TIDM) of three years duration presents with weight loss. She has normal menstrual cycles and bowel habits are unchanged. Her EMT is 23 kg/in2 and lab studies reveal « haemoglobin of 7 g/dL (11.5-16.5) and a MCV of 69 £L (80-96). The most lkely diggnosis is ) Anoresia nervosa ) Bacterial overgrowth ©) Beta-thalassaemia minor JS © 4) Coeliac disease ©) Crohn’s disease Question Explanation: Coeliac disease is the likely option as this patient has autoimmnne disease (IDM), an iron deficiency anaemia and litle in the why of symptoms, Bacterial overgrowth is associated with profuse diarrhoea and a macrocytosis due to vitarrin B12 consuraption. Crohn's disease agein would be expected to be symptomatic. ‘Mark this question & => Question 29 of 30 A.54 year old type 2 diabetic male presents for anmual review. His glycaemic control is sub-optimal on diet alone and his most recent HbA Ic is 7.9% (3.8-6.4). You elect to treat him with metformin 500 mg twice daily. The most appropriate interval to re-check his HbA 1c would be which one of the fallowing? a) Two weeks b) One month c) Two-three months 4) Four-sis months ) Six-tweive months Question Explanation: The HbA ¢ is arellection of the glycosylation of the haemoglobin moeity by glucose, There is a strong correlation between the alycosylation of this molecule and average plasma glucose concentrations hence ts widespread use in clinical practice as a tool to assess dycaemic control, Furthermore, studies reveal its prognostic significance in both microvascular and macrovascular risk. The life span of the red cellis 120 days. HbAlc reflects average blood glucose levels daring the halflife of the red cell (about 60 days) and so the recommended appropriate interval for re-measuring HbA.1e following change in therapy is two mouths, ‘Mark this question & => Question 29 of 30 A.54 year old type 2 diabetic male presents for anmual review. His glycaemic control is sub-optimal on diet alone and his most recent HbA Ic is 7.9% (3.8-6.4). You elect to treat him with metformin 500 mg twice daily. The most appropriate interval to re-check his HbA 1c would be which one of the fallowing? a) Two weeks b) One month ¥ © ¢) Two-three months 4) Four-sis months ) Six-tweive months Question Explanation: The HbA ¢ is arellection of the glycosylation of the haemoglobin moeity by glucose, There is a strong correlation between the alycosylation of this molecule and average plasma glucose concentrations hence ts widespread use in clinical practice as a tool to assess dycaemic control, Furthermore, studies reveal its prognostic significance in both microvascular and macrovascular risk. The life span of the red cellis 120 days. HbAlc reflects average blood glucose levels daring the halflife of the red cell (about 60 days) and so the recommended appropriate interval for re-measuring HbA.1e following change in therapy is two mouths, ‘Mark this question & Question Td : 212620 Question 30 of 30 A47 year old women noted that a "small hanp" moved up and down in the lower part ofler neck whenever she swallowed. On exam itis confismed thet she has a single, firm, 2 cm thyroid nodule in the right lobe. There are no other abnormalities. Her pulse is 82min and regular. TSH is within nomal limits. The most appropriate next step in management is which one of the following? a) Clincal observation, repeating the TSH atleast once a year b) CT scan of the thyroid c) Determination of T3 and 14 levels 4) Radioactive iodine RAD thyroid scan e) Fine needle aspiration (FINA) for cytology of the mass Question Explanation: Most thyroid nodules are benign. However, # the serum thyroid-stinulating hormone (TSH) is high or normal, the nodule could be neoplastic. Thus, fine needle aspiration (FMLA) should be done to rule out cancer. Management based upon FINA results are as fellow: + Malignant: Surgery is the next best step. + Benign: Observe the patient clinically. + Indeterminate or suspicious; Radioactive iodine uptake (RAIU) scan is indicated (D) to determnine whether the nodule is “hot" or "cold! Ge, whether ittakes up iodine or net, respectively). A hot nodule is unlikely to he malignant, and ifitis not causing hypeathyroidism, it can be left alone and observed clinically (A). A cold nodale is more concerning for malignancy, however, and surgical resection should follow. Iris important to note that most cold nodules are benign. CT scan of the thyroid gland (B) ‘3 important in the workup of a patient who has a goiter and is complaining of obstructive symptoms, such as shoriness of ‘breath or cifficuk, swallowing Tthas no role in the initial workup of a thyroid nodule Tayroid nadales can be benign but hyperfinctioning (toxic adenoma); therefore, thyroid function must be detsrminsd, This petient has a normal TSH, so further pursuit of T3 and T4 (C) is unnecessary. ‘Mark this question & Question Td : 212620 Question 30 of 30 A.47 year old women noticed that a ‘small ump" moved up and down in the lower part ofher neck whenever ske swallowed, On exam itis confirmed that she has a single, firm, 2 cm thyroid nodule in the night lobe, There are no other abromnalities, Her pulse is 82/min and regular. TSH is within normal limits. The most appropnate next step in management is which one of the following? a) Clincal observation, repeating the TSH atleast once a year b) CT scan of the thyroid ) Determination of 73 and T4 levels d) Radioactive iodine (RAD thyroid scan Y © ¢) Fine needle aspiration (FNA) for cytology of the mass Question Explanation: Most thyroid nodules are benign. However, # the serum thyroid-stinulating hormone (TSH) is high or normal, the nodule could be neoplastic. Thus, fine needle aspiration (FMLA) should be done to rule out cancer. Management based upon FINA results are as fellow: + Malignant: Surgery is the next best step. + Benign: Observe the patient clinically. + Indeterminate or suspicious; Radioactive iodine uptake (RAIU) scan is indicated (D) to determnine whether the nodule is “hot" or "cold! Ge, whether ittakes up iodine or net, respectively). A hot nodule is unlikely to he malignant, and ifitis not causing hypeathyroidism, it can be left alone and observed clinically (A). A cold nodale is more concerning for malignancy, however, and surgical resection should follow. Iris important to note that most cold nodules are benign. CT scan of the thyroid gland (B) ‘3 important in the workup of a patient who has a goiter and is complaining of obstructive symptoms, such as shoriness of ‘breath or cifficuk, swallowing Tthas no role in the initial workup of a thyroid nodule Tayroid nadales can be benign but hyperfinctioning (toxic adenoma); therefore, thyroid function must be detsrminsd, This petient has a normal TSH, so further pursuit of T3 and T4 (C) is unnecessary. ‘Mark this question => Question Td : 51256 Question 1 of 30 ‘Whatis the drug choice for an obese diabetic type 2 patient? a) Acarbose b) Metformin ¢) Rosiglitazone ) Glyburide Anowor [UERHSREREN) omer User's Explanation Report An Eiror Question Explanation: Motformin has teen clearly established as the drag of choice in obese petionts with diabetes mellitus. Metformin is as effective a sulphongslureas at reducing Hb AIC, and most importantly has a beneficial effect on overall mortality in obese patients Metformin monotherapy is unlikely to be effective in patients who fail to respond to sulphorylureas, but in patients who are secondary failures to sulphonylureas the addition of metformin causes substantial blood gtucose lowering, Metformin should be avoided in patients with renal dysfunction. Otherwice its effects on bodyweight, serum lipids and ts lack of hypoglycemia effect make i an excellent first ine agent. ‘Mark this question => Question Td : 51256 Question 1 of 30 ‘What is the drug choice for an obese diabetic type 2 pation? a) Acarbose v¥ © b) Metformin ¢) Rosiglitazone ) Glyburide Anowor [UERHSREREN) omer User's Explanation Report An Eiror Question Explanation: Motformin has teen clearly established as the drag of choice in obese petionts with diabetes mellitus. Metformin is as effective a sulphongslureas at reducing Hb AIC, and most importantly has a beneficial effect on overall mortality in obese patients Metformin monotherapy is unlikely to be effective in patients who fail to respond to sulphorylureas, but in patients who are secondary failures to sulphonylureas the addition of metformin causes substantial blood gtucose lowering, Metformin should be avoided in patients with renal dysfunction. Otherwice its effects on bodyweight, serum lipids and ts lack of hypoglycemia effect make i an excellent first ine agent. 2242014 3:02:01 PM ‘Mark this question & => Question Td : 51559 Question 2 of 30 A.27 year old female presents with history of fatigue, weight gain, menstrual irregularity and declining memory for three months. ‘Which laboratory result is Ulely in the diagnosis of the underlying problem? a) TSH increased, free T4 increased b) TSH decreased, free T3 increased c) TSH increased, free T4 decreased 4) TSH decreased, flee T3decreased ¢) TSH within normal range, free T4 within normal range Question Explanation: Hypothyroidism is common, especially among older people, particularly women; it affects about 10% of older women Itcan, however, occur at any age. Very severe hypothyroidism is called myxedema. Insufficient thyroid hormone causes body functions to slow. Symptoms are subtle and develop gradually. They may be mistalcen for depression, especially among older people Facial expressions become dull, the voice is hoarse and speechis slow, eyelids droop, and the eyes and face become pufly. Many people with hypothyroidism gain weight, become constipated, and are unable to tolerate celd, The har becomes sparse, coarse, and dry, and the skin becomes coarse, dry. scaly, and thick. Usually hypethyroidism can be diagnosed with one simple blood test: the measurement of thyroid. stimulating hormone, which will be high due to alack of negative feedbacle. A second blood testis needed to sneasure the level of the thyroid honmone T4 that is not bound by protein (Bee). A low level confinns the diagnosis of hypothyroidism, 2242014 3:02:01 PM ‘Mark this question & => Question Td : 51559 Question 2 of 30 A.27 year old female presents with history of fatigue, weight gain, menstrual irregularity and declining memory for three months. ‘Which laboratory result is Ulely in the diagnosis of the underlying problem? a) TSH increased, free T4 increased b) TSH decreased, free T3 increased ¥ © c) TSHincreased, free T4 decreased A) TSH decreased, free T3decreased ¢) TSH within normal range, free T4 within normal range Question Explanation: Hypothyroidism is common, especially among older people, particularly women; it affects about 10% of older women Itcan, however, occur at any age. Very severe hypothyroidism is called myxedema. Insufficient thyroid hormone causes body functions to slow. Symptoms are subtle and develop gradually. They may be mistalcen for depression, especially among older people Facial expressions become dull, the voice is hoarse and speechis slow, eyelids droop, and the eyes and face become pufly. Many people with hypothyroidism gain weight, become constipated, and are unable to tolerate celd, The har becomes sparse, coarse, and dry, and the skin becomes coarse, dry. scaly, and thick. Usually hypethyroidism can be diagnosed with one simple blood test: the measurement of thyroid. stimulating hormone, which will be high due to alack of negative feedbacle. A second blood testis needed to sneasure the level of the thyroid honmone T4 that is not bound by protein (Bee). A low level confinns the diagnosis of hypothyroidism, ‘Marke this question e& => Question Ta : 54419 Question 3 of 30 A 44 year old woman is brought to the Emergency Department after she fainted in a supermarket. According to her hushand she was diagnosed as "hypoglycemic" by a physician 2 years ago, The criteria that is most in favor of the diagnosis of hypoglycemia is a) Low blood glucose while symptomatic ) Abnomnal 5 hour glucose tolerance test ©) Typical prodrome of tachycardia 6) History of post-prandial syncope Question Explanation: Hypoglycemia umelated to exogenous insulin therapy is an uncommon clinical syndrome characterized by low plasma glucose level, symptomatic sympathetic nervous system stimulation, and CNS dysfunction. Many drugs and disorders cause it. AT? hour fast performed in a controlled setting is the standard for diagnosis. Patients drink only noncaloric, noncaffeinated ‘beverages, and plasma glucese is measured at baceline, whenever symptoms occur, end q 4 to 6 hor q 1 to 2 hif glucose falls below 60 mg/dL (3. 3mmotL). Serum insulin, C-peptide, and proinsulin should be measured at times of hypoglycemia to cistinguish endogenous from exogenous (factitious) hypoglycemia, The fact is terminated at 72 hifthe patient has experienced no symptoms and glucose remaine normal, sooner if glucose decreases to S45 mg/dL. (¢ 2.5 mmoVL) in the presence of hypoglycemic symptoms Report An Error ‘Marke this question e& => Question Ta : 54419 Question 3 of 30 A 44 year old woman is brought to the Emergency Department after she fainted in a supermarket. According to her hushand she was diagnosed as "hypoglycemic' by « physician 2 years ago. The criteria that is most in faver of the diggnosis of hypoglycentia is 2) Low blood glucose while symptomatic Y © &) Abnomal 5 hour glucose tolerance test ©) Typical prodrome of tachycardia 6) History of posi-prandial syncope Question Explanation: Hypoglycemia umelated to exogenous insulin therapy is an uncommon clinical syndrome characterized by low plasma glucose level, symptomatic sympathetic nervous system stimulation, and CNS dysfunction. Many drugs and disorders cause it. AT? hour fast performed in a controlled setting is the standard for diagnosis. Patients drink only noncaloric, noncaffeinated ‘beverages, and plasma glucese is measured at baceline, whenever symptoms occur, end q 4 to 6 hor q 1 to 2 hif glucose falls below 60 mg/dL (3. 3mmotL). Serum insulin, C-peptide, and proinsulin should be measured at times of hypoglycemia to cistinguish endogenous from exogenous (factitious) hypoglycemia, The fact is terminated at 72 hifthe patient has experienced no symptoms and glucose remaine normal, sooner if glucose decreases to S45 mg/dL. (¢ 2.5 mmoVL) in the presence of hypoglycemic symptoms Report An Error Mark this question => Question Id : 55432 Question 4 of 30 ‘A. 36 year old female presents with palpitations, restlessness, sweating, weight loss, and a tremor for the past 6 months. Exemination shows a fine tremor, lid lag and stare, and pretibial myzedema Tae thyroid gland is diffusely enlarged, asymmetric, and lobular. A ‘onut is present over the gland, Investigations show an undetectable level of thyroid stimulating hormone, an increased lewel of thyroid kormones, and aa increased radioactive iodine uptace. Tae diagnosis of Graves disease is made and the treatment cptions ere discussed. The patient selects radioactive iodine therapy. This patent is at greatest isk for which of the following? 2) Granulocytopenia ) Hypothyroidism ) Recurrent laryngeal nerve damage Thyroid carcinoma Anower [BIIRIBEN) ter veors Explana Question Explanation: Hypothyroidism is this main complication of radioactive indine therapy, affecting up to 70% of patients in 10 years. Radioactive iodine therepy is a safe and effective treatment for Greves's disease because it can provide the same ablative effects of surgery ‘without the surgical complications. There is no evidence that this treatment increases the risk for carcinoma. Report An Error Mark this question => Question Id : 55432 Question 4 of 30 ‘A. 36 year old female presents with palpitations, restlessness, sweating, weight loss, and a tremor for the past 6 months. Exemination shows a fine tremor, lid lag and stare, and pretibial myzedema Tae thyroid gland is diffusely enlarged, asymmetric, and lobular. A ‘onut is present over the gland, Investigations show an undetectable level of thyroid stimulating hormone, an increased lewel of thyroid kormones, and aa increased radioactive iodine uptace. Tae diagnosis of Graves disease is made and the treatment cptions ere discussed. The patient selects radioactive iodine therapy. This patent is at greatest isk for which of the following? 2) Granulocytopenia Y © b) Hypothyroidism ) Recurrent laryngeal nerve damage Thyroid carcinoma Anower [BIIRIBEN) ter veors Explana Question Explanation: Hypothyroidism is this main complication of radioactive indine therapy, affecting up to 70% of patients in 10 years. Radioactive iodine therepy is a safe and effective treatment for Greves's disease because it can provide the same ablative effects of surgery ‘without the surgical complications. There is no evidence that this treatment increases the risk for carcinoma. Report An Error ‘Mark this question = => Question Td : 57079 Question 5 of 30 A AB-year-old has marked shortness of breath that deteriorated over the last two weeles. He has a hard, irregular thyroid mass with no retrostemal extension and he has some diffcuity breathing. He appears dlinivally euthyroid, Diagnose! a) Bleed into a thyroid nodule ») Follicular thyroid carcinoma ©) Anaplastic carcinoma of thyroid 4) Medullary thyroid carcinoma ¢) Multino dular goitre Answer | Eaplanaion Other User's Explanation Report An Error Question Explanation: ‘This pationt i likely to have anaplastic carcinoma oftthe thyroid with comprescion/infibration of the trachea, Thic is ualicely to be a smuttinodatar goiter as this would be unlikely to compress the trachea unless retrosternal and the description is more compatiole with a thyroid malgnancy. Medullary thyroid cancer is associated mest often with MEN Il and would be particularly umasual. Again follcviar would be unlleely to produce such marked infiltrative features in such a short peciod ofttme. Typically patients with follicular disease would present with a nodule and/or LAP. ‘Mark this question = => Question Td : 57079 Question 5 of 30 A. 48-year-old has merked shortness of breath that deteriorated aver the last two weeks. He has a hard, irregular thyroid mass with xno retrostemal extension and he has some difficuily breathing. He appears clinically culhyroid. Diagnose! a) Bleed into a thyroid nodule ) Folicular thyroid carcinoma JY © ©) Anaplastic carcinoma of thyreid 4) Medullary thyroid carcinoma €) Multinodular goitre Answer | Eaplanaion Other User's Explanation Report An Error Question Explanation: ‘This pationt i likely to have anaplastic carcinoma oftthe thyroid with comprescion/infibration of the trachea, Thic is ualicely to be a smuttinodatar goiter as this would be unlikely to compress the trachea unless retrosternal and the description is more compatiole with a thyroid malgnancy. Medullary thyroid cancer is associated mest often with MEN Il and would be particularly umasual. Again follcviar would be unlleely to produce such marked infiltrative features in such a short peciod ofttme. Typically patients with follicular disease would present with a nodule and/or LAP. 2124/2014 3:02:47 PM ‘Mark this question => Question Id : 59571 Question 6 of 30 A patent with ascites is suspected to have secondary hyperaldosteronism, The typical levels of electrolytes in an aliquot specimen of urine would be a) Sodium 2 mEg/L, potassium 40 mEq/L ») Sodium 5 mEq/L, potassium 0 mEq/L «) Sodium 40 mEq/L, potassinm 40 mEq/L @) Sodium 89 mEg/L, potassinm 2 mEq(L «) Sodium 100 mEq/L, potassium 20 mEq/L Anewor (UBARESHEH) ner veers Explanation Report An Exot Question Explanation: Secondary hyperaldosteronism is characterized by sodium retention, and thas decreased urinary sodium excretion, while potassium secretion is normal to increased, 2124/2014 3:02:47 PM ‘Mark this question => Question Id : 59571 Question 6 of 30 A patent with ascites is suspected to have secondary hyperaldosteronism, The typical levels of electrolytes in an aliquot specimen of urine would be Y © a) Sodium 2 mEq/L, potassium 40 mEq. ») Sodium 5 mEq/L, potassium 0 mEq/L «) Sodium 40 mEq/L, potassinm 40 mEq/L @) Sodium 89 mEg/L, potassinm 2 mEq(L «) Sodium 100 mEq/L, potassium 20 mEq/L Anewor (UBARESHEH) ner veers Explanation Report An Exot Question Explanation: Secondary hyperaldosteronism is characterized by sodium retention, and thas decreased urinary sodium excretion, while potassium secretion is normal to increased, 2124/2014 3:02:58 PM ‘Mark this question & => Question Td : 60451 Question 7 of 30 ‘Which one of the following is NOT indicated in the management of thyrotoxic crisis? a) Cooling blankets bb) Hydrocortisone c) Thyroxine 4) Propytthiouracil @) Iodide Answer | Bopanation | Other User's Explanation Report An Error Question Explanation: Thyrotoxic crisis is a medical emergency ands an exagerated thyrotoxocosis ot thyroid storm, Thyroxine would worsen the condition, but iodide will block thyroid hormone secretion. Cooling blankets help lessen the hyperpyrezia, and hydrocortisone is useful in the management of the stress and relative adrenal in-sufficiency. Propythiouracil inhibits the synthesis of thyroid hormone. 2124/2014 3:02:58 PM ‘Mark this question & => Question Td : 60451 Question 7 of 30 ‘Which one of the following is NOT indicated in the management of thyrotoxic crisis? a) Cooling blankets b) Hydrocortisone Y © c) Thyroxine 4) Propylthiouracil 6) Iodide Answer | Bopanation | Other User's Explanation Report An Error Question Explanation: Thyrotoxic crisis is a medical emergency ands an exagerated thyrotoxocosis ot thyroid storm, Thyroxine would worsen the condition, but iodide will block thyroid hormone secretion. Cooling blankets help lessen the hyperpyrezia, and hydrocortisone is useful in the management of the stress and relative adrenal in-sufficiency. Propythiouracil inhibits the synthesis of thyroid hormone. Mark this question => Question Id : 62730 Question 8 of 30 A.57 year old womanis scheduled for an abdominal operation. She has hypotuyroidism that is controled with thyroid replacement medication and will be unable to eat or drink for 4 days following the procedure. She is concemed about receivirg her thrrcid medication, What should be advised to her? 2) Although she will have symptoms of hypotyroidism. She will not be given the medication ') Although she will not receive the medication, she will have no adverse effects ©) The medication will be administered daily while the nasogastric tube is clamped 4) The medication will be discontinued temporarly before the operation €) She will receive the medication intravenously Anower (UBIIRIRRN) oterucorsFxplanation Report An Error Question Explanation: ‘Thyroxine is the hormone of choice for replacement therapy. Ti has a halflife of seven days, and any alteration in dose is not reflected for four to six weeks. Therefore, itis very unlikely that she will develop signs and symptoms of hypethyrcidism. She will not be given the medication either through the nasogastric tube or intravenously, nor does she require a preoperative loading dose. Mark this question => Question Id : 62730 Question 8 of 30 A.57 year old woman is scheduled for an abdominal operation. She has hypothyroidism that is controlled with thyroid replacemeat medication and will be unable to eat or drink for 4 days following the procedure, She is concemed about receiving her thyrcid medication, What should be advised to her? a) Although she will have symptoms of hypothyroidism. She will not be given the medication Y © bd) Although she will not receive the medication, she will have no adverse effects ©) The medication will be administered daily while the nasogastric tube is clamped 4) The medication will be discontinued temporary before the operation 2) She will receive the medication intravenously Anower (UBIIRIRRN) oterucorsFxplanation Report An Error Question Explanation: ‘Thyroxine is the hormone of choice for replacement therapy. Ti has a halflife of seven days, and any alteration in dose is not reflected for four to six weeks. Therefore, itis very unlikely that she will develop signs and symptoms of hypethyrcidism. She will not be given the medication either through the nasogastric tube or intravenously, nor does she require a preoperative loading dose. 2i24.2014 3:03:23 PM ‘Mark this question & => Question Ii : 76096 Question 9 of 30 Hyperosmolar norketotc diabetic coma is most common in a) The elderly +b) Patients with a history of ketoacidosis. ©) Adolescents 4) Pregnant women. €) Children, Avewor [EQIRNEHER) otner Usore Explanation Repost An Eror Question Explanation: This condition is most commonly seen in the elderly. Ibis usually found in diabetics between the ages of 50 and 70. It is not more common ia patients with previous episodes of ketoacidosis, adolescents, pregnant women, or children. 2i24.2014 3:03:23 PM ‘Mark this question & => Question Ii : 76096 Question 9 of 30 Hyperosmolar norketotc diabetic coma is most common in ¥ © a) The elderly +b) Patients with a history of ketoacidosis. ©) Adolescents 4) Pregnant women. €) Children, Avewor [EQIRNEHER) otner Usore Explanation Repost An Eror Question Explanation: This condition is most commonly seen in the elderly. Ibis usually found in diabetics between the ages of 50 and 70. It is not more common ia patients with previous episodes of ketoacidosis, adolescents, pregnant women, or children. 3582, ‘Mark this question & => Question Td : Question 10 of 30 A.56 year old female presents with difficulty swallowing A thorough workup by a gastroenterologist fails to reveal any primary esophageal disease. The woman is referred to an endocrinologist. A biopsy of the thyroid reveals the infitretion with fibrous tissue. The mest probable diagnocis ic which of the following? a) Eyeball protrusion 'b) Maccive and tender thyroid gland ) Painfil and tender thyroid gland 4) Single large thyroid nodule ©) Very fiom thyroid gland Question Explanation: Riedel thyroiditis, also called ligneous (rocklike) stroma, is a rare form of chronic: thyroiditis characterized microscopically by a marked fibrous reaction that destroys most or all ofthe thyroid gland and may involve adjacent structures. The etiology is unknown. Clinically, this disease tends to affect middle-aged and older, mostly female patients and canses the thyroid to have a fim "woody" texture, Itmay be clisically mistalcen for a neck malignancy and can cause symptoms of stridor, dyspnea, dysphesia, laryageal nerve paralysis, or hypothyroidism. Eyeball protrusion (choice A) suggests the hyperthyroidism of Graves's disease. A, soft thyroid gland (choice B) suggests ‘attinodular goiter ‘A painful and tender thyroid (Choice C) suggests subacute granulomatous (de Quervain) thyroicltis ‘A single large thyroid nodule (choice D) could be due to either a thyroid adenoma or thyroid cancer. 3582, ‘Mark this question & => Question Td : Question 10 of 30 A.56 year old female presents with difficulty swallowing A thorough workup by a gastroenterologist fails to reveal any primary esophageal disease The woman is referred to an endocrinologist. A biopsy of the thyroid reveals the infiltration with fibrous tissue The mest probable diagnocis ic which of the following? a) Eyeball protrusion 'b) Maccive and tender thyroid gland ) Painfil and tender thyroid gland 4) Single large thyroid nodule Y © ©) Very firm thyroid gland Question Explanation: Riedel thyroiditis, also called ligneous (rocklike) stroma, is a rare form of chronic: thyroiditis characterized microscopically by a marked fibrous reaction that destroys most or all ofthe thyroid gland and may involve adjacent structures. The etiology is unknown. Clinically, this disease tends to affect middle-aged and older, mostly female patients and canses the thyroid to have a fim "woody" texture, Itmay be clisically mistalcen for a neck malignancy and can cause symptoms of stridor, dyspnea, dysphesia, laryageal nerve paralysis, or hypothyroidism. Eyeball protrusion (choice A) suggests the hyperthyroidism of Graves's disease. A, soft thyroid gland (choice B) suggests ‘attinodular goiter ‘A painful and tender thyroid (Choice C) suggests subacute granulomatous (de Quervain) thyroicltis ‘A single large thyroid nodule (choice D) could be due to either a thyroid adenoma or thyroid cancer. Mark this question & => Question Ti : 83666 Question 11 of 30 A. 16 year old patient is taken to a physician because of severe episodic headaches, accompanied by perspiration and palpitations. ‘The patient is experiencing headache at the tme of the examination and his blood pressure is 175/125 mm Hg with regular heart rate of 90, Treatment with phenoxybenzamine relieves his symptoms. The investigation thet would be most helpful for establishing the agnosis is 8) Serum albumin ) Serum cortisol ©) Serum renin 6) Urinary Bence-Jones proteins ¢) Urinary vanilyimandelis acid (MA) Anower [UERPNSHON) ote: User's Exptanai Question Explanation: The suspected tumor is pheochromecytoma, 10% of which ocour in children. This tumor arises from neural crest cells ofthe adrenal medulla, The symptoms » (paroxysmal hypertension, palptations, anziety) are produced when the tumor secretes epinephrine, norepinephtine, and other vaso active amines into the circulation, The diagnosis can be established with plasma catecholamine concentrations or concentrations of the norepinephrine metabolte VIMLA in a 24-hour urine, The latter offers the advantage of providing a longer tine sample so that the intermittent secretion is more likely to be picked up, Serum albumin (choice A) can be low m lver and renal disease, but is unaffected in pheochromocytoma Cortisol (choice B) excess is seen with Cushing syndrome. Renin (choice C) excess is associated with renal disease, secondary hyperaldosteronism, renal cell carcinoma, and renal ischemia, Bence-Jones proteins (Choice D) are a marker for mubiple myeloma, representing urinary excretion of myeloma light chains. Report An Error Mark this question & => Question Ti : 83666 Question 11 of 30 A 16 year old patient is taken to a physician because of severe episodic headaches, accompanied by perspiration and palpitations. ‘The patient ic experiencing headache at the time of the exemination and hie blood presewe is 175/125 mm Hg with regular heart sate of 90, Treatment with phenoxybenzamnine relieves his symptoms. The investigation thet would be most helpful for establishing the agnosisis 8) Serum albumin ) Serum cortisol 6) Serum renin 6) Urinary Bence-Tones proteins oY © ©) Urinary vanillyimandelic acid (WMA) Anower [UERPNSHON) ote: User's Exptanai Question Explanation: The suspected tumor is pheochromecytoma, 10% of which ocour in children. This tumor arises from neural crest cells ofthe adrenal medulla, The symptoms » (paroxysmal hypertension, palptations, anziety) are produced when the tumor secretes epinephrine, norepinephtine, and other vaso active amines into the circulation, The diagnosis can be established with plasma catecholamine concentrations or concentrations of the norepinephrine metabolte VIMLA in a 24-hour urine, The latter offers the advantage of providing a longer tine sample so that the intermittent secretion is more likely to be picked up, Serum albumin (choice A) can be low m lver and renal disease, but is unaffected in pheochromocytoma Cortisol (choice B) excess is seen with Cushing syndrome. Renin (choice C) excess is associated with renal disease, secondary hyperaldosteronism, renal cell carcinoma, and renal ischemia, Bence-Jones proteins (Choice D) are a marker for mubiple myeloma, representing urinary excretion of myeloma light chains. Report An Error ‘Mark this question & => Question Td : 92182 Question 12 of 30 A 24-year old thin woman feels weak with diaphoresis, weight loss, insomnia, and menstrual abnormalities. She is tachycardic, with moist skin and a iremor along hypemreflexia TSH is <0.1 yIU/ml. Thyroid is tender and radioactive iodine uptake is low. She has a low-grade temperature. Diagnose: a) Struma ovat b) Thyrotozicosis factitia ©) Grave's disease D Subacute thyroiditis ©) Autonomous toxic adenomas of the thyroid Answer (Bevinaton) Other User's Explanation Report An Error Question Explanation: Subacute thyroiditis is usualy secondary to a viral infection. The thyroid is moderately enlarged and tender with a decrease in the radioactive iodine uptake. Analgesics are given for the pain and fever, and this disease resolves on its own overtime. Preformed thyroid hormones are seleaced from the follicles. Struma ovatiis thyroid tive located in ovarian dermoid tumors and teratemas which autonomously secrete thyroid hormone. Thyrotoxicesis facia is caused by ingestion of excessive amounts of exogenous thyroid hormones. ‘This can also be caused by consuraptions of ground beef contaminated with bovine thyro:d gland. Grave's disease is the most common cause of thyrotoxicosis. Ikis an autoimmune disorder. The radioactive iodine uptake is increased and the thyroid gand is enlarged, Autonomous toxic adencmac of the thyroid can be single or mubiple. There ic inoreace in the radioactive iodine uptake and negative anti-thyroid antibodies in the plasma. ‘Mark this question & => Question Td : 92182 Question 12 of 30 A 24-year old thin woman feels weak with diaphoresis, weight loss, insomnia, and menstrual abnormalities. She is tachycardic, with moist skin and a tremor along hyperrellexia TSH is <0.1 yilU/ml, Thyroidis tender and radioactive iodine uptake is low. She has a low-grade temperature. Diggnoss: a) Struma ovat +) Thyrotoxicosis factitia ©) Grave's disease Y © & Subacute thyroiditis ©) Autonomous toxic adenomas of the thyroid Answer (Bevinaton) Other User's Explanation Report An Error Question Explanation: Subacute thyroiditis is usualy secondary to a viral infection. The thyroid is moderately enlarged and tender with a decrease in the radioactive iodine uptake. Analgesics are given for the pain and fever, and this disease resolves on its own overtime. Preformed thyroid hormones are seleaced from the follicles. Struma ovatiis thyroid tive located in ovarian dermoid tumors and teratemas which autonomously secrete thyroid hormone. Thyrotoxicesis facia is caused by ingestion of excessive amounts of exogenous thyroid hormones. ‘This can also be caused by consuraptions of ground beef contaminated with bovine thyro:d gland. Grave's disease is the most common cause of thyrotoxicosis. Ikis an autoimmune disorder. The radioactive iodine uptake is increased and the thyroid gand is enlarged, Autonomous toxic adencmac of the thyroid can be single or mubiple. There ic inoreace in the radioactive iodine uptake and negative anti-thyroid antibodies in the plasma. ‘Mark this question. & => Question Tid : 95843 Question 13 of 30 Mixed metabolic acidosis and alkalosis is most likely seen in the setting of which of the following? ) Severe pulmonary disease ) Lactic acidosis, €) Diabetic acidosis and lactic acidosis. 4) Diabetic acidosis and vomiting €) Mechanical ventilation Question Explanation: An individual with diabetic acidosis and vomiting is most likely to have a mixed metabolic acidosis and alkalosis. The vomiting expels hydrochloric acid from the stomach, producing a metabolic alkalosis. Severe pulmonary disease prodaces metabolic alkalosis and respiratory acidosis, Laciic acidosis produces a pure metabolic acidosis. Lactic acidosis combined with diabetic acidosis similarly produces metabolic acidosis. Mechanical ventilation can result in respiratory alkalosis. ‘Mark this question. & => Question Tid : 95843 Question 13 of 30 Mixed metabolic acidosis and alkalosis is most likely seen in the setting of which of the following? ) Severe pulmonary disease ) Lactic acidosis, €) Diabetic acidosis and lactic acidosis. Y © 4) Diabetic acidosis and vomiting €) Mechanical ventilation Question Explanation: An individual with diabetic acidosis and vomiting is most likely to have a mixed metabolic acidosis and alkalosis. The vomiting expels hydrochloric acid from the stomach, producing a metabolic alkalosis. Severe pulmonary disease prodaces metabolic alkalosis and respiratory acidosis, Laciic acidosis produces a pure metabolic acidosis. Lactic acidosis combined with diabetic acidosis similarly produces metabolic acidosis. Mechanical ventilation can result in respiratory alkalosis. ‘Marke this question e& => Question Td : 100160 Question 14 of 30 4.20 year oldman who presented with complaints decrease in libido, decrease in facial and body hair, and poor muscle strength was diagnosed as having primary hypogonadism, FALGG statement regarding primary hypogonadism is a) Luteinizing hoamone and follicle stimulating hormone are elevated, +b) Serum testosterone concentrations are low. ) Primary hypogonadism can lead to osteeporosis 4) There is deficiency of gonadowopin-releasing hormone fiom the hypothalamus. ©) Men with primary hypogonadism require lifelong androgen replacement: Question Explanation: Secondary hypogonadism, not primary hypogonadism, is associated with a deficiency of gonadotropin releasing hormone from the hypothalamus or a deficiency of LH and FSH from the pituitary gland, Secondary hypogonadism can be dus to pituitary adenoma, craniopharyngioma, or intracranial radiation. Serum LH and FSH levels are elevated in primary hypogonadism because of the lack of testosterone production in the gonads, Serum testosterone levels are low secondary to decreased production in the scrotum, Lack of testosterone can lead to a decrease in erythropoiesis, osteoporotis, and muscle mass. Lifelong androgen replacement in the form of intramuscular testosterone and transdermal testosterone patches are needed in primary hypogonadism, ‘Marke this question e& => Question Td : 100160 Question 14 of 30 4.20 year oldman who presented with complaints decrease in libido, decrease in facial and body hair, and poor muscle strength was diagnosed as having primary hypogonadism, FALGG statement regarding primary hypogonadism is a) Luteinizing hoamone and follicle stimulating hormone are elevated, +b) Serum testosterone concentrations are low. ) Primary hypogonadism can lead to osteeporosis Y © 4) Thereis deficiency of gonadotropin-releasing hormone fiom the hypothalamus. ©) Men with primary hypogonadism require lifelong androgen replacement: Question Explanation: Secondary hypogonadism, not primary hypogonadism, is associated with a deficiency of gonadotropin releasing hormone from the hypothalamus or a deficiency of LH and FSH from the pituitary gland, Secondary hypogonadism can be dus to pituitary adenoma, craniopharyngioma, or intracranial radiation. Serum LH and FSH levels are elevated in primary hypogonadism because of the lack of testosterone production in the gonads, Serum testosterone levels are low secondary to decreased production in the scrotum, Lack of testosterone can lead to a decrease in erythropoiesis, osteoporotis, and muscle mass. Lifelong androgen replacement in the form of intramuscular testosterone and transdermal testosterone patches are needed in primary hypogonadism, ‘Mark this question & => Question Td : 107449 Question 15 of 30 A.47 year old man has symptoms of reftactory hypertension, hypokalemia, and artericlar narrowing on fnduscopic exam. His blood pressure is not controlled by anthypertensives, No abdominal bnuit or hypecpiamentation was observed. Investigations revealed a low placena renin level. The most lcely diagnosis ic a) Primary hyperaldosteronism ) Secondary hyperaldosteronicen c) Renal artery stenosis 4) Pheochromocytoma ©) Cushing's disease Question Explanation: Symptoms of primary aldosteronism are caused by excessive aldosterone secretion from the adrenal cortex either ftom adrenal hyperplasia os an adrenal adenoma Continued elevated aldosterone levels cause hypokalemia and diastolic hypertension Diuretics exacerbate the symptoms of kypokalemia, Renin levels are usualy suppressed Secondary hyperaldosteronistn resuits from, stimulation ofthe adrenal glomerulesa by extra adrenal factors. The renin levelis usually elevated. Renal artery stenosis is another possible cause for secondary hypertension. This patient has no abdominal bruits 10 suggest renal artery stenosis. The potassium level would be normal, notlow. Pheochromocytoma patients would present with palpitations, fluctuating blood pressure levels, and diaphoresis. The potassiam level is usualy normal. The CT scan may detect the tamor in the adrenal medulla Cushing's’ disease rwould result in hypertension secondary to exiceas corticosteroids in the body. This disorder is more conamoa in femnales. The patient would also be obese with osteoporosis, striae, and moon faces. ‘Mark this question & => Question Td : 107449 Question 15 of 30 A47 year old manhas symptoms of reftactory hypertension, hypokalemia, and artericlar narrowing on funduscopic exam. His blood pressure is not controlled by anthypertensives, No abdominal bnuit or hypecpiamentation was observed. Investigations revealed a low plasma renin level. The most Hicely diagnosis is Y © a) Primary hyperaldosteronism b) Secondary hyperaldesteronism. c) Renal artery stenosis d) Pheochromocytoma 2) Cushing's disease Question Explanation: Symptoms of primary aldosteronism are caused by excessive aldosterone secretion from the adrenal cortex either ftom adrenal hyperplasia os an adrenal adenoma Continued elevated aldosterone levels cause hypokalemia and diastolic hypertension Diuretics exacerbate the symptoms of kypokalemia, Renin levels are usualy suppressed Secondary hyperaldosteronistn resuits from, stimulation ofthe adrenal glomerulesa by extra adrenal factors. The renin levelis usually elevated. Renal artery stenosis is another possible cause for secondary hypertension. This patient has no abdominal bruits 10 suggest renal artery stenosis. The potassium level would be normal, notlow. Pheochromocytoma patients would present with palpitations, fluctuating blood pressure levels, and diaphoresis. The potassiam level is usualy normal. The CT scan may detect the tamor in the adrenal medulla Cushing's’ disease rwould result in hypertension secondary to exiceas corticosteroids in the body. This disorder is more conamoa in femnales. The patient would also be obese with osteoporosis, striae, and moon faces. 2242014 3:04:50 PM Mark this question & => Question Td : 109759 Question 16 of 30 A black obese female of age 52 years presents with new onset polyuria, polydipsia, weakness, and fatigue. Her fasting gucose level is 301 mg/dl A family history of diabetes mellitus is present, Which of the following statement is FALSE? 2) Type II diabetes is strongly inherited and complications are usually present at diagnosis. ') The resuite of the diabetes control and complications trial show that a decrease in HbA Ic by 2% will result in 60% less retinopathy ©) Diabetesis a cardiovascular risk factor. 6) The goal should be to control glucose, lower cholestorel (LDL) below 130 mg/dl, and keep the HbAle below 10%. ©) 60-70% ofall diabetics in the Unted States have no insulin dependent diabetes mellitus Answer ( Expianation) Other User's Explanation Report An Error Question Explanation: The goalin dicbetes control should be to keep the hemoglobin A1c level below 8%. By the time one presents with symptoms of type Tr diaketes, some end organ damage has heen done. The diabetes contyol and complications tral has heen one of the largest Glahetes trails in the nation. I showed that tight control of glucose will resul in slow progressicn of retinopathy and nephropathy. Diabetes is a strong risk factor for myocardial infarction, stroke, peripheral vascular disease, and generalized atherosclerosis. Three to five percent of the United States population has diabetes. Steen million people (diegnosed and undagnosed) have diabetes. Diabetes is two to five times more common in minorities. Sixty to seventy percent of all diabetics are non inculn dependent and 10% are juverile oncet diabetics 2242014 3:04:50 PM Mark this question & => Question Td : 109759 Question 16 of 30 A black obese female of age 52 years presents with new onset polyuria, polydipsia, weakness, and fatigue. Her fasting gucose level is 301 mg/dl A family history of diabetes mellitus is present, Which of the following statement is FALSE? 2) Type II diabetes is strongly inherited and complications are usually present at diagnosis. ') The resuite of the diabetes control and complications trial show that a decrease in HbA Ic by 2% will result in 60% less retinopathy ©) Diabetesis a cardiovascular risk factor. Y © 6) The goal should be to control glucose, lower cholesterol (LDL) below 130 mg/dl, and keep the HbAle below 10%. ©) 60-70% ofall diabetics in the Unted States have no insulin dependent diabetes mellitus Answer ( Expianation) Other User's Explanation Report An Error Question Explanation: The goalin dicbetes control should be to keep the hemoglobin A1c level below 8%. By the time one presents with symptoms of type Tr diaketes, some end organ damage has heen done. The diabetes contyol and complications tral has heen one of the largest Glahetes trails in the nation. I showed that tight control of glucose will resul in slow progressicn of retinopathy and nephropathy. Diabetes is a strong risk factor for myocardial infarction, stroke, peripheral vascular disease, and generalized atherosclerosis. Three to five percent of the United States population has diabetes. Steen million people (diegnosed and undagnosed) have diabetes. Diabetes is two to five times more common in minorities. Sixty to seventy percent of all diabetics are non inculn dependent and 10% are juverile oncet diabetics ‘Mark this question = => Question Td : 119179 Question 17 of 30 A-woman aged 77 years presents with weakness, fatigue, anorexia, and symptoms of salt craving. Her blood pressure was 80/60 mmHg en examination and she had hyperpigmentation. Investigations ceveal hyperkalemia, The most licely diagnocis is a) Chronic primary adrenocostical insufficiency 1b) Hypereninenic hypoaldesteronisea c) Cushing's disease 4) Conn's eyadrome e) Secondary adrenal insufficiency Answer | botanation Other User's Explanation Report An Error Question Explanation: Chronic primary adrenocortical insufficiency is associated with hyperkalemia, aypemreninemia, and hypotension. The hyperpigmentationis secondary to excessive ACTH production from the pinitary gland. Hyporeainemic hypoaldosteronism causes hyperchloreric metabolic acidosis. There ic mineralocortiooid deficiency and low renin concentrations, ar well as cevere hyperkalemia Cushing's disease is secondary to excess cortisol levels. This can be from an elevation of ACTH fom the pinitary gland Con's smdrome is primary hyperaldosteronism. This disease causes Lypertension, hypernatremia, and hypokelemia Secondary adrenal insufficiency is from inadequate stimulation of the adrenal cortex by ACTH. Clinically significant mineralocorticoid deficioncy is rare, Hyperlcalemia and hyperpigmentation are rare ‘Mark this question = => Question Td : 119179 Question 17 of 30 A-woman aged 77 years presents with weakness, fatigue, anorexia, and symptoms of salt craving. Her blood pressure was 80/60 mmHg en examination and she had hyperpigmentation. Investigations ceveal hyperkalemia. The most licely diagnocis is Y © a) Chronic primary adrenocortical insufficiency 1b) Hypereninenic hypoaldesteronisea c) Cushing's disease 4) Conn's eyadrome e) Secondary adrenal insufficiency Answer | botanation Other User's Explanation Report An Error Question Explanation: Chronic primary adrenocortical insufficiency is associated with hyperkalemia, aypemreninemia, and hypotension. The hyperpigmentationis secondary to excessive ACTH production from the pinitary gland. Hyporeainemic hypoaldosteronism causes hyperchloreric metabolic acidosis. There ic mineralocortiooid deficiency and low renin concentrations, ar well as cevere hyperkalemia Cushing's disease is secondary to excess cortisol levels. This can be from an elevation of ACTH fom the pinitary gland Con's smdrome is primary hyperaldosteronism. This disease causes Lypertension, hypernatremia, and hypokelemia Secondary adrenal insufficiency is from inadequate stimulation of the adrenal cortex by ACTH. Clinically significant mineralocorticoid deficioncy is rare, Hyperlcalemia and hyperpigmentation are rare 2124/2014 3:05:12 PM Mark this question >_> Question 18 of 30 Aninfertle female aged 22 years who has cligomenorthea end gelactorrhea with a prolactin level of 1,157 ng/ml and low esirogen level is diagnosed as having hyperprolactnemia. Which of the following is NOT a cause of typerprolactinemria? a) Microadenoma of the pinuitary gland. ') Empty sella syndrome ) Phenothiazines. 4) Sasccidosis of the pituitary and kypothalarmas. €) Bromocriptine Question Explanation: Bromocriptine is a dopamine agonist, Side effects include nausea, vomiting, and mood changes. I: inhibits prolactin secretion in all forms of prolactinemia Galactorthea is elitnincted and cyclic menses and fertility may return without complete normalization of prolactin levels. Micrcadenoma and macroadencma of the pituitary gland are the most common causes of hyperprolactinemia. Surgical resection results ina cure, Relapse ofthe tumor may occur Empty sella syndrome is diagnosed by CT or MRI of the brain, 2124/2014 3:05:12 PM Mark this question >_> Question 18 of 30 Aninfertle female aged 22 years who has cligomenorthea end gelactorrhea with a prolactin level of 1,157 ng/ml and low esirogen level is diagnosed as having hyperprolactnemia. Which of the following is NOT a cause of typerprolactinemria? a) Microadenoma of the pinuitary gland. ') Empty sella syndrome ) Phenothiazines. 4) Sasccidosis of the pituitary and kypothalarmas. Y © €) Bromocriptine Question Explanation: Bromocriptine is a dopamine agonist, Side effects include nausea, vomiting, and mood changes. I: inhibits prolactin secretion in all forms of prolactinemia Galactorthea is elitnincted and cyclic menses and fertility may return without complete normalization of prolactin levels. Micrcadenoma and macroadencma of the pituitary gland are the most common causes of hyperprolactinemia. Surgical resection results ina cure, Relapse ofthe tumor may occur Empty sella syndrome is diagnosed by CT or MRI of the brain, 2242014 3:05:30 PM Mark this question & => Question 19 of 30 In nondiabetics, hypoglycemia does NOT resub fiom which of the following? a) Alcohol. 6) lasulin ©) Cushing's syndrome. ©) Postprandial ©) Sarcomas. Question Explanation: Causes of hypoglycemia in nondiabetics may also produce hypoglycemia in diabetics. A wide variety of drugs may produce hypoglycemia. Alcohol inhibits hepatic uconeogenesis. Hypoglycemia occurs when hepatic gluconeogenesis is the main source of ghicose production, such as during periods of starvation. Tnsulin and oral hypoglycemics may produce hypoalycemia when these ruge are used surreptitiously for ulterior motives. In addition to innulinomas, aon beta cell tumors may produce hypoglycemia. These tumors suppress plasma insulin levels and, in some cases, induce hypoglycemia through massive glucose utilization. Reactive, postprandial hypoglycemia occurs in persons with rapid gastric emptying, such as following a gastrectomy. Rapid absorption of ghuicose stimulates brick insulin release, leading to appropriately elevated inculin levels. Cushing's syndrome results from glucocorticoid excess. Hyperglycemia is a characteristic feature Report An Error 2242014 3:05:30 PM Mark this question & => Question 19 of 30 In nondiabetics, hypoglycemia does NOT resub fiom which of the following? 2) Alcohol. ) Insulin V¥ © ©) Cushing’s syndrome. ©) Postprandial ©) Sarcomas Question Explanation: Causes of hypoglycemia in nondiabetics may also produce hypoglycemia in diabetics. A wide variety of drugs may produce hypoglycemia. Alcohol inhibits hepatic uconeogenesis. Hypoglycemia occurs when hepatic gluconeogenesis is the main source of ghicose production, such as during periods of starvation. Tnsulin and oral hypoglycemics may produce hypoalycemia when these ruge are used surreptitiously for ulterior motives. In addition to innulinomas, aon beta cell tumors may produce hypoglycemia. These tumors suppress plasma insulin levels and, in some cases, induce hypoglycemia through massive glucose utilization. Reactive, postprandial hypoglycemia occurs in persons with rapid gastric emptying, such as following a gastrectomy. Rapid absorption of ghuicose stimulates brick insulin release, leading to appropriately elevated inculin levels. Cushing's syndrome results from glucocorticoid excess. Hyperglycemia is a characteristic feature Report An Error 2i24/2014 3:05:46 PM ‘Mark this question & => Question Id : 178115 Question 20 of 30 A.15 year old boy presents because of a tender breast "lamp" that he noticed a couple of weeks ago. He is on the football team and recently began taking testosterone to make his "muscles thicker." Exam shows a 2 cm area of palpable breast issue on the leftand a | cm area of palpable breast tissue on the nght. The most appropriate next step is 2) Discontiauation of testosterone b) Evaluation for liver disease ¢) Fine-needle aspiration d) Mammography 2) Reassurance Answer | Explanation | Other User's Explanation —_ Report An Error Question Explanation: ‘This patient has gynecomastia, which is most lcely due to his testosterone intake. Testosterone is converted to estradiol in extraglancular tissues and leads to feminization. Gynecomastia is yew common in adolescents during puberty and is frequently asymmetric and tender. It usually regreccec before age 20. Evaluation for liver disease ie unnecessary at thi point because the gynecomastia is most likely due to his testosterone usage or to puberty. He has no other signs ofliver disease. Fine aeedle aspiration and mammography are used to evaluzte a dominant breast mess or possibly gmecomastia in a patient who is not going through puberty who has anegative drug history or who has a rapidly growng. large (>4-cm) mass. This patient has explanations for gynecomastia (tectosterone and puberty), making firther workup too aggressive at this time. Reassurance would be the appropriate answer if this boy were not taking testosterone. 2i24/2014 3:05:46 PM ‘Mark this question & => Question Id : 178115 Question 20 of 30 A.15 year old boy presents because of a tender breast "lamp" that he noticed a couple of weeks ago. He is on the football team and recently began taking testosterone to make his "muscles thicker." Exam shows a 2 cm area of palpable breast issue on the leftand a | cm area of palpable breast tissue on the nght. The most appropriate next step is Y © a) Discontinuation of testosterone b) Evaluation for liver disease ¢) Fine-needle aspiration d) Mammography 2) Reassurance Answer | Explanation | Other User's Explanation —_ Report An Error Question Explanation: ‘This patient has gynecomastia, which is most lcely due to his testosterone intake. Testosterone is converted to estradiol in extraglancular tissues and leads to feminization. Gynecomastia is yew common in adolescents during puberty and is frequently asymmetric and tender. It usually regreccec before age 20. Evaluation for liver disease ie unnecessary at thi point because the gynecomastia is most likely due to his testosterone usage or to puberty. He has no other signs ofliver disease. Fine aeedle aspiration and mammography are used to evaluzte a dominant breast mess or possibly gmecomastia in a patient who is not going through puberty who has anegative drug history or who has a rapidly growng. large (>4-cm) mass. This patient has explanations for gynecomastia (tectosterone and puberty), making firther workup too aggressive at this time. Reassurance would be the appropriate answer if this boy were not taking testosterone. 2i24/2014 3:05:59 PM Mark this question e& => Question Td : 178973 Question 21 of 30 ‘A patiant reports periodic bouts of ightheadedness and confision, His fasting blood glacose is 45 mg/dL. Plasma insulin is found to tbe markedly elevated, but plasma C peptide is undetectable, The most likely explanation for these findings is which one of the following? a) Dumping syndrome b) Factitious hypoglycemia ¢) Growth hormone deficiency 4d) Insulinoma ¢) Pheochromocytoma Answer (Explanation | Other User's Explanation Report An Error Question Explanation: Factitious hypogiycetria, or self-induced hypoalycemia, is a psychological disorder that results from surreptitious self injection with insulin Tris most commonly seen in healthcare professionals or in diabetic patients or their relatives. The key to diagnosing this, disorder is that injectable insulin preparations do not contain any C-peptide, Hence, the injected insulin (which explains the increased plasma insulin concentration) produces hypoglycemia. The hypoglycemia then suppresses beta-cell secretion of endogenous insulin and C-peptide, Dumping syndrome can produce a non-fasting hypoglycemia, This is a reactive hypoglycemia produced by excessive secretion of insulin after rapid discharge of ingested carbohydrate into the small intestine in patients with previons gastrectomy. The excessive insulin secretion may be clue to overstimulation of parasympathetic reflexes or io increased secretion of insulnctropic gut peptides. The increase in insuln secretion would be accompanied by an increase in C-peptide secretion, Patients with growth honnone deficiency can also experience hypoglycemic episodes, especially when fasting. Growth hormone has an anti-insulin action, and when this hormone is deficient, the ability of insulin to induce hypoglycemia is increased. Growth hormone deficiency could not explain the increase in plasma insulin and decrease in plasma C peptide. Insulimoma also produces tasting hypoglycemia cue to uncontrolled secretion of insulin by a beta-cell tumor. However, in this case, plasma insuln and C-peptide will both be increased, since the two polypeptides are secreted by beta cells in @ one to one molar ratio, Pheochromocyioma results from a catecholamine- secreting tumer. IF anything, blood glucose is increared in this disorder. The excess catecholamines stimulate glycogenclysis and alse suppress insulin secretion. 2i24/2014 3:05:59 PM Mark this question e& => Question Td : 178973 Question 21 of 30 A patient reports periodic bouts of ightheadedness and confusion. His fasting blood glacose is 45 mg/dL. Plasma insulin is found to the markedly elevated, but plasma C peptide is undetectable, The most likely explanation for these fincings is which one of the following? a) Dumping syndrome ¥ © b) Factitous hypoglycemia ©) Growth hormone deficiency ) Insalinoma ©) Pheochromocytoma Answer (Explanation | Other User's Explanation Report An Error Question Explanation: Factitious hypogiycetria, or self-induced hypoalycemia, is a psychological disorder that results from surreptitious self injection with insulin Tris most commonly seen in healthcare professionals or in diabetic patients or their relatives. The key to diagnosing this, disorder is that injectable insulin preparations do not contain any C-peptide, Hence, the injected insulin (which explains the increased plasma insulin concentration) produces hypoglycemia. The hypoglycemia then suppresses beta-cell secretion of endogenous insulin and C-peptide, Dumping syndrome can produce a non-fasting hypoglycemia, This is a reactive hypoglycemia produced by excessive secretion of insulin after rapid discharge of ingested carbohydrate into the small intestine in patients with previons gastrectomy. The excessive insulin secretion may be clue to overstimulation of parasympathetic reflexes or io increased secretion of insulnctropic gut peptides. The increase in insuln secretion would be accompanied by an increase in C-peptide secretion, Patients with growth honnone deficiency can also experience hypoglycemic episodes, especially when fasting. Growth hormone has an anti-insulin action, and when this hormone is deficient, the ability of insulin to induce hypoglycemia is increased. Growth hormone deficiency could not explain the increase in plasma insulin and decrease in plasma C peptide. Insulimoma also produces tasting hypoglycemia cue to uncontrolled secretion of insulin by a beta-cell tumor. However, in this case, plasma insuln and C-peptide will both be increased, since the two polypeptides are secreted by beta cells in @ one to one molar ratio, Pheochromocyioma results from a catecholamine- secreting tumer. IF anything, blood glucose is increared in this disorder. The excess catecholamines stimulate glycogenclysis and alse suppress insulin secretion. 2242014 3:06:13 PM ‘Mark this question & => Question Id : 195681 Question 22 of 30 A 65 year old Caucasian male is overweight and takes exercise. He has been treated for hypertension for 5 years andis controlled on. Smg of Ramipril, He also takes 20mg of Simvastatin for hypercholesterolaemia A 75g oral giucose tolerance test gave a result consistent with impaired glucose tolerance (IGT) with a 2 hour plasma glucose concentration of 9.3mmolfL (3.0-6.0). He is keen to know what would be his risk of developing type Zciabetes. What do you tell him? 8) 6A over 6 years b) 10% over 6 years ©) 33% over 6 years 4) 60% over years 2) 100% over years newer [ETRRIAN) ner user's Explanation Report An Exot Question Explanation: Individuals with IGT are at significant risk of progression to type 2 diabetes. A number of studies have looked at the absolute risk of progression fiom IGT to type 2 diabetes. The large and widely quoted Hoom study which looked at 1342 Caucasian non diabetic subjects found that 33.8% progressed to type 2 diabetes over 6 years follow up. This increased to 54 5% if individuals had both IGT and IFG. A similar rate of progression for individuals with IGT was Vaccaro who studied a Caucasian group in Taly. Intensive lifestyle changes involving diet changes, regular exercice and weightloss have been shown to reduce the rate of progression to type 2 diabetes. 2242014 3:06:13 PM ‘Mark this question & => Question Id : 195681 Question 22 of 30 A.65 year old Caucasian male is overweight and takes exercise. He has been treated for hypertension for 5 years and is controlled on Srng of Ramipril He alsa talees 20mg af Simvastatin for hypercholesteralaemia. A 75g oral ghicase tolerance test gave a result consistent with impaired glucese tolerance (GT) with a 2 hour plasma glacose concentration of 9. 3mmolL (3.0-6.(). He is keen to know what would be his risk of developing type 2ciabetes. What do you tell him? 8) 6% over 6 years 6) 10% over 6 years JS © 2) 33% over 6 years 4) 60% over years 2) 100% over years newer [ETRRIAN) ner user's Explanation Report An Exot Question Explanation: Individuals with IGT are at significant risk of progression to type 2 diabetes. A number of studies have looked at the absolute risk of progression fiom IGT to type 2 diabetes. The large and widely quoted Hoom study which looked at 1342 Caucasian non diabetic subjects found that 33.8% progressed to type 2 diabetes over 6 years follow up. This increased to 54 5% if individuals had both IGT and IFG. A similar rate of progression for individuals with IGT was Vaccaro who studied a Caucasian group in Taly. Intensive lifestyle changes involving diet changes, regular exercice and weightloss have been shown to reduce the rate of progression to type 2 diabetes. Mark this question e& => Question Td : 195985 Question 23 of 30 ATI year old diabetic male with poor glycaemic control has a history of two previous myocardil infarctions, and gets exertional angna at 50 yards. He has diabetic maculopatiy. and distal sensory neuropatly. His current treatment includes : metformin 500 mg three times a day, gimepiride 4 mg daly, insulin deter 20 unts at night, Perindopril @ mg every day, furosemide 80 mg daly, agpirin 75 mg daily and atorvastatin 20 mg daily. HbA1c is 9.2%, fasting giucose is 13.4 mmol/L and creatinine is 130 pmol/L. LFTs are normal. Which strategy is appropriate for his lycaemic control? 8) Add prandial insulin (eg NovoRapid) three times daily +) Add premixed insulin (eg humalog 25) twice daily and stop Lantus ) Adil rosiglitazone 4 mg daily 4) Substitute metformin with Avandamet 4/500 mg twice daily €) Up titrate the dose ofinsulin deternie snower [REGIRIRNBAY) Other User's Explana Question Explanation: This patient has uncontrolled glycaemia despite the current dose of Insulin gargine, oral hypoglycaemic therapy and dietary intervention Rosighiazone is already conirandiceted because of the history of heart failure and the use of insulin. The current basal insulin regime of the insuln anelogue detemir is failing to control his glycaemia; however the cutrent dose is inadequate, Carrent practice would favour increasing the dose of detemir aiming for a fasting (pre-breakfast) BM of <7.0, Only once fasting readings of this level are achieved, (with a sub optimal HbA 1c), would one thine of adding a prandial insulin. The caveat would be nocturnal bypogycaemia, in which case a chance the insulin regime would be warranted, however this was not mentioned in the vignette Report An Error Mark this question e& => Question Td : 195985 Question 23 of 30 ATI year old diabetic male with poor glycaemic control has a history of two previous myocardil infarctions, and gets exertional angna at 50 yards. He has diabetic maculopatiy. and distal sensory neuropatly. His current treatment includes : metformin 500 mg three times a day, gimepiride 4 mg daly, insulin deter 20 unts at night, Perindopril @ mg every day, furosemide 80 mg daly, agpirin 75 mg daily and atorvastatin 20 mg daily. HbA1c is 9.2%, fasting giucose is 13.4 mmol/L and creatinine is 130 pmol/L. LFTs are normal. Which strategy is appropriate for his lycaemic control? 8) Add prandial insulin (eg NovoRapid) three times daily +) Add premixed insulin (eg humalog 25) twice daily and stop Lantus ) Adil rosiglitazone 4 mg daily 4) Substitute metformin with Avandamet 4/500 mg twice daily Y © ©) Up titrate the dose ofinsulin detemir anower [REGIBIRNBAY) Other User's Explana Question Explanation: This patient has uncontrolled glycaemia despite the current dose of Insulin gargine, oral hypoglycaemic therapy and dietary intervention Rosighiazone is already conirandiceted because of the history of heart failure and the use of insulin. The current basal insulin regime of the insuln anelogue detemir is failing to control his glycaemia; however the cutrent dose is inadequate, Carrent practice would favour increasing the dose of detemir aiming for a fasting (pre-breakfast) BM of <7.0, Only once fasting readings of this level are achieved, (with a sub optimal HbA 1c), would one thine of adding a prandial insulin. The caveat would be nocturnal bypogycaemia, in which case a chance the insulin regime would be warranted, however this was not mentioned in the vignette Report An Error ‘Mark this question & => Question Td : 195995 Question 24 of 30 A.33 year old female is being investigated for tntus. Her MRI scanis normal except for a ptuiary tumour of U.9cm confined to the pituitary fossa. Thyroid fimction tests, prolactin, LH, FSH and estradiol concentrations are all normal. The most appropriate management approach for this patient would be a) Pimitary biopsy ) Reasaure and continued observation ©) Transphenoidal kypophysectomy 4) Treat with dopamine agonist therapy €) Stereotactic pituitary irradiation Anwor [REIRIRNBA) othe: User's Explanation Report An Esror Question Explanation: This patient has a coincidentally noted pituitary tumor, has no endocrine symptoms and appears to have normal endocrine function although we are act provided with information pestaning to cortisol secretory function nor GH. With this caveat in mind, the most appropriate strategy would be observation and repeat scenning ‘Mark this question & => Question Td : 195995 Question 24 of 30 A 33 year old female is being investigated for tnntus. Her MRI scanis normal except for a ptutary tumour of U.9cm confined to the pibitary fossa. Thyroid function tests, prolactin, LH, FSH and estradiol concentrations are all ncrmal, The most appropriate management approach for this patient would be a) Pimitary biopsy Y © b) Reassure and contimed observation ¢) Transphenoidal kypophysectomy. 4) Treat with dopamine agonist therapy ¢) Stereotactic pituitary irradiation Anwor [REIRIRNBA) othe: User's Explanation Report An Esror Question Explanation: This patient has a coincidentally noted pituitary tumor, has no endocrine symptoms and appears to have normal endocrine function although we are act provided with information pestaning to cortisol secretory function nor GH. With this caveat in mind, the most appropriate strategy would be observation and repeat scenning Mark this question e& => Question Td : 196129 Question 25 of 30 A 53 year old male with a history of dyslipidaemia and hypertension attends for 2 75 g oral ghicose tolerance test (OGTT) as pat of his cardiovascular risk assessment and screening for Type 2 diabetes. He is overweight with a BMI of 29 kea/m? his BP is 135/85 mmmFTg on a combination of arnlodipine and Perindopei. His venous plasma OGTT result is as follows: [Dmimtes [6.3 120 minutes||10.4) ‘The correct interpretation of these results is which one of the following? @) Tnpaited fasting glucose (FG) +) Impaired fasting glucose and impaired glucose tolerance ©) Impaired glacose tolerance (GT) 46) Normal glucose tolerance ©) Type 2 diabetes ee Question Explanation: The World Health Orgarisation recotrmend the use of the OGTT for the diagnosis of Type 2 diabetes as it incorporates the fasting and two hour post glucose load which can identify different subgroups of diabetic patients and different categories of individuals at risk of Type 2 diabetes. The diagnostic criteria are lasma ghicose fasting (mmol) 2 hour post ghicose (nmol/L) Diabeticl> 7.0 fp 11.1 fer |<70 [8-110 Fe _l[e1-6.3)<78 Ti the absence of marked hyperglycaemia and symptoms such as weigh loss, polyuria or pelydipsia these results should be confirmed, by a repeat test on another day. Mark this question e& => Question Td : 196129 Question 25 of 30 A 53 year old male with a history of dyslipidaemia and hypertension attends for 2 75 g oral ghicose tolerance test (OGTT) as pat of his cardiovascular risk assessment and screening for Type 2 diabetes. He is overweight with a BMI of 29 kea/m? his BP is 135/85 mmmFTg on a combination of arnlodipine and Perindopei. His venous plasma OGTT result is as follows: [Dmimtes [6.3 120 minutes||10.4) ‘The correctiinterpretation of these results is which one of the following? a) Impaired fasting glucose (EG) Y © b) Impaired fasting glucose and impaired glucose tolerance ©) Impaired glicose tolerance (GT) 6) Normal glucose tolerance ©) Type 2 diabetes ee Question Explanation: The World Health Orgarisation recotrmend the use of the OGTT for the diagnosis of Type 2 diabetes as it incorporates the fasting and two hour post glucose load which can identify different subgroups of diabetic patients and different categories of individuals at risk of Type 2 diabetes. The diagnostic criteria are lasma ghicose fasting (mmol) 2 hour post ghicose (nmol/L) Diabeticl> 7.0 fp 11.1 fer |<70 [8-110 Fe _l[e1-6.3)<78 Ti the absence of marked hyperglycaemia and symptoms such as weigh loss, polyuria or pelydipsia these results should be confirmed, by a repeat test on another day. [2124/2014 3:07:09 PM “Mark this question es Question 26 of 30 Question Id : 196161 A 25 year old woman with no previous history of diabetes presents with afirst episode of diabetic ketoacidosis. There is no evidence of infection but she recently started a new medication. Tae drug that is implicated in preciptitating diabetic ketoacidosis is which one of the following? a) Olanzapine b) Omeprazole ©) Progestogen only contraceptive pill Sodium valproate ©) Venlafaxine Anower [UBQRRESEN) otner veers Explanation Report An Error Question Explanation: The atypical antipsychctics euch as Olanzapine have been implicated in precipitating diabetes as well as diabetic ketoacidosis. Other rugs unplicated include thiazide diuretics, beta sympathomimetics and steroids, [2124/2014 3:07:09 PM “Mark this question es Question 26 of 30 Question Id : 196161 A 25 year old woman with no previous history of diabetes presents with afirst episode of diabetic ketoacidosis. There is no evidence of infection but she recently started a new medication. Tae drug that is implicated in preciptitating diabetic ketoacidosis is which one of the following? Y © @) Olanzapine b) Omeprazole ©) Progestogen only contraceptive pill Sodium valproate ©) Venlafaxine Anower [UBQRRESEN) otner veers Explanation Report An Error Question Explanation: The atypical antipsychctics euch as Olanzapine have been implicated in precipitating diabetes as well as diabetic ketoacidosis. Other rugs unplicated include thiazide diuretics, beta sympathomimetics and steroids, ‘Mark this question & => Question Ii : 196467 Question 27 of 30 A 34 year oldis noted to have ++ alycosuria, Her BMI is 35 kglin2 and fasting plasma glucose is 7.4 minol (3.0-6.0). The measure that would be most effective in reducing her insulin resistance is which one of te allowing? a) Gibenclanide ) Inculia c) Metformin 4) Weight loss ¢) Repaglinide Answer | baplanation Other User's Explanation Report An Error Question Explanation: ‘The most appropriate treatment of this obese female with type 2 diabetes, which would also substantially reduce insulin concentrations, is weight loss. Gibenclamide may be associated with increased insulin resistance as stimulates insulin secretion, as docs repaglinide, Metformin would inprove insulin sensitivity but would aot do it as nm as weight less. The unfortunate thing is that weight loss is the most difficuk strategy for the patientto adhere 10. ‘Mark this question & => Question Ii : 196467 Question 27 of 30 A 34 year oldis noted to have ++ alycosuria, Her BMI is 35 kglin2 and fasting plasma glucose is 7.4 minol (3.0-6.0). The measure that would be most effective in reducing her insulin resistance is which one of te allowing? a) Gibenclanide ) Inculia c) Metformin Y © Ad) Weight loss ¢) Repaglinide Answer | baplanation Other User's Explanation Report An Error Question Explanation: ‘The most appropriate treatment of this obese female with type 2 diabetes, which would also substantially reduce insulin concentrations, is weight loss. Gibenclamide may be associated with increased insulin resistance as stimulates insulin secretion, as docs repaglinide, Metformin would inprove insulin sensitivity but would aot do it as nm as weight less. The unfortunate thing is that weight loss is the most difficuk strategy for the patientto adhere 10. 2242014 3:07:37 PM ‘Mark this question & => Question Ti : 196609 Question 28 of 30 A patient is receiving treatment with recombinant human growth hormone (GE). A recognized side effect of GH therapy is which one of the following? a) Benign intra-cranial hypertension (BIH) ») Melanoma c) Osteoporosis 4) Prolongation of the QT interval 2) Prostatic hypertrophy Answer | Exsianaion Other User's Explanation Report An Error Question Explanation: GH is rerely associated with BIH; the mecharism probably is related to fluid retention The commonest side effect of GH therapy is uid retention, though other side effects inciude gynaecomactia, hypertension and atria fibrilation Benign prostatic hyperplasia (BPH) as not been reported, 2242014 3:07:37 PM ‘Mark this question & => Question Ti : 196609 Question 28 of 30 A patient is receiving treatment with recombinant human growth hormone (GE). A recognized side effect of GH therapy is which one of the following? Y © a) Benign intra-cranial hypertension (BIH) b) Melanoma c) Osteoporosis d) Prolengation of the QT interval @) Prostatic hypertrophy Answer | Exsianaion Other User's Explanation Report An Error Question Explanation: GH is rerely associated with BIH; the mecharism probably is related to fluid retention The commonest side effect of GH therapy is uid retention, though other side effects inciude gynaecomactia, hypertension and atria fibrilation Benign prostatic hyperplasia (BPH) as not been reported, ‘Marie this question & => Question Ta : 196660 Question 29 of 30 “Which finding would be expected in active acromegaly with associated diabetes mellitus? a) Diabetes mellitus is due to an auto-immune process b) Growth hormone concentrations arc suppressed with hyperglycaemia c) Growth hormone concentrations are suppressed with hyperglycaemia ) There is insulin resistance ¢) Treatment with a somatostatin analogue is contra-indicated newer (UEIRNRI) omer cers Explanation Repost An Error Question Explanation: Insulin resistance stems from the excessive growth hormone (GH) concentrations (anti-innulin effects) that of course fail to suppress with hyperglycaemia. Acromegaly is often effectively treated with somatostatin anclogues which may improve glycaemic control, Many ofthe effects of GH. are mediated through IGE- 1, concentrations of which are high in acromegaly. Diabetes melltus is due to the inculn resistance and is not due to avto-iteanune insulitis, ‘Marie this question & => Question Ta : 196660 Question 29 of 30 “Which finding would be expectedin active acromegaly with associated diabetes mellitu:? a) Diabetes mellius is due to an aute-irarmune process +b) Growth hormone concentrations are suppressed with hyperglycaemia c) Growth hormone concentrations are suppressed with hyperglycaemia Y © A) There is insulin resistance ¢) Treatment with a somatostatin analogue is contra-indicated newer (UEIRNRI) omer cers Explanation Repost An Error Question Explanation: Insulin resistance stems from the excessive growth hormone (GH) concentrations (anti-innulin effects) that of course fail to suppress with hyperglycaemia. Acromegaly is often effectively treated with somatostatin anclogues which may improve glycaemic control, Many ofthe effects of GH. are mediated through IGE- 1, concentrations of which are high in acromegaly. Diabetes melltus is due to the inculn resistance and is not due to avto-iteanune insulitis, ‘Mark this question & Question Td : 217119 Question 30 of 30 A 49-year-old hypertensive has shooting pains in his hands worse at night He has gained a great deal of weight over the past couple of years His appearance is changing with time. He has increased thirst and urination. There is out of proportion enlargement of the hands and feet, What test to be conducted first? a) Cortisol level b) CT scan of brain ©) Fasting glucose level 4) Growth hormone levels ¢) Inculincike growth factor newer [UERIRIRIN oer users Explanation Report An ror Question Explanation: This patiert has acromegaly, which is an excess of growth hormone. This is evidenced by increased hand and foot size and coarse facial features with protruding jaw. Other features, including tooth space widening and macroglossia, can present with acromegaly. Impaired glucose tolerance often exists, as does diabetes. Hypertension is present in one-third of patients with acromegaly. Complaints of headache and visual fields loss can occur. Patients may have carpal tunnel smcrome. The test or choice to diagmose this condition is insulin-lice growth actor, which is elevated to 5 times the upper limit of normal. Another test to be used to diagnose acromegaly is measurement of growth hormone after administration of 100 grams or glucose. In healthy patient the growth hormone level would be suppressed, whereas the levels in patient >5 times the upper limit ofnormal. Another test that could be used to diagnose acromegaly is to obtain a growth hormone level after the administration of gincose 100 g. Ina healthy patient the growth hormone level would be suppressed, whereas in a patient with acromegaly, the level would remain high (I.e., >5 ng/mL) Cortisol level might be helpful in diagnosing Cushing’s syncrome, which can also be associated with insulin sesistance, hypertension, and weight gain. However, Cushing's syndrome does not Share other features associated with acromegaly, CT scan of the brain would likely teveal a pituitary macroadenoma A CT scan (or other imaging technique) should be performed to confirm most endocrine disorders only after they are suggested serologically. This isto avoid diagnostic errors that can be made with the presence oftumors discovered incidentally. The presence of such tumors, which are Unrelated to the patient's condition, could potentially lead the Clinician down an incorrect course. Cushing's syndrome can be associated with insulin resistance, hypertension, and weight gain but doesn't share other features associated with acromegaly. (CT scan of the brain would licely reveal a macroadenoma in this patient, A CT scanning (or other imaging technique) should be conducted to confirm most endocrine disorders only after they are suggested serologically. This is to avoid diagnostic errors that can be made due to the presence of tumors incidentally discovered that are unrelated to the patient's concition and potentially lead the clinician to an incosrest diagnosis and course of treatment, Fasting glucose level is the best screening test for ciaberes. Tris lely that the patient has diabetes secondary to acromegaly, evidenced by polyuria and polydipsia Itis appropriate to measure fasting dlucose levels in this patient but itis not the test to use to diagnose the patient's underlying medical problem. Growth hormone levels should not be used as a screening meshanism to diagnose acromegaly because gonadotropin-releasing hosmone and growth hormone fuctuete normally throughout the day. Exercise, or even ecute diness, can contribute to such fluctuations. Falsely elevated growrh hormone levels, therefore, can be found in patients with normal growth hormone levels. imilarle natente with arramecaty ran have falee narmal orawth hermane lewele ‘Mark this question & Question Td : 217119 Question 30 of 30 A.49-year-old hypertensive kas shooting pains in his hands worse at right. He has gained a great deal of weight over the past couple ofyears: His appearance is changing with time. He has increased thirst and urination. There is out of proportion enlargement of the hhands and feet. What test to be conducted first? a) Cortisol level b) CT scan of brain c) Fasting glucose level 4) Growth hormone levels Y © 2) Insulin-like growth factor newer [UERIRIRIN oer users Explanation Report An ror Question Explanation: This patiert has acromegaly, which is an excess of growth hormone. This is evidenced by increased hand and foot size and coarse facial features with protruding jaw. Other features, including tooth space widening and macroglossia, can present with acromegaly. Impaired glucose tolerance often exists, as does diabetes. Hypertension is present in one-third of patients with acromegaly. Complaints of headache and visual fields loss can occur. Patients may have carpal tunnel smcrome. The test or choice to diagmose this condition is insulin-lice growth actor, which is elevated to 5 times the upper limit of normal. Another test to be used to diagnose acromegaly is measurement of growth hormone after administration of 100 grams or glucose. In healthy patient the growth hormone level would be suppressed, whereas the levels in patient >5 times the upper limit ofnormal. Another test that could be used to diagnose acromegaly is to obtain a growth hormone level after the administration of gincose 100 g. Ina healthy patient the growth hormone level would be suppressed, whereas in a patient with acromegaly, the level would remain high (I.e., >5 ng/mL) Cortisol level might be helpful in diagnosing Cushing’s syncrome, which can also be associated with insulin sesistance, hypertension, and weight gain. However, Cushing's syndrome does not Share other features associated with acromegaly, CT scan of the brain would likely teveal a pituitary macroadenoma A CT scan (or other imaging technique) should be performed to confirm most endocrine disorders only after they are suggested serologically. This isto avoid diagnostic errors that can be made with the presence oftumors discovered incidentally. The presence of such tumors, which are Unrelated to the patient's condition, could potentially lead the Clinician down an incorrect course. Cushing's syndrome can be associated with insulin resistance, hypertension, and weight gain but doesn't share other features associated with acromegaly. (CT scan of the brain would licely reveal a macroadenoma in this patient, A CT scanning (or other imaging technique) should be conducted to confirm most endocrine disorders only after they are suggested serologically. This is to avoid diagnostic errors that can be made due to the presence of tumors incidentally discovered that are unrelated to the patient's concition and potentially lead the clinician to an incosrest diagnosis and course of treatment, Fasting glucose level is the best screening test for ciaberes. Tris lely that the patient has diabetes secondary to acromegaly, evidenced by polyuria and polydipsia Itis appropriate to measure fasting dlucose levels in this patient but itis not the test to use to diagnose the patient's underlying medical problem. Growth hormone levels should not be used as a screening meshanism to diagnose acromegaly because gonadotropin-releasing hosmone and growth hormone fuctuete normally throughout the day. Exercise, or even ecute diness, can contribute to such fluctuations. Falsely elevated growrh hormone levels, therefore, can be found in patients with normal growth hormone levels. imilarle natente with arramecaty ran have falee narmal orawth hermane lewele

You might also like